M

Chiudi

Esercizi sui numeri complessi volume 2 (per matematica o fisica)

Esercizi misti Numeri Complessi, Fondamenti

Home » Esercizi sui numeri complessi volume 2 (per matematica o fisica)

Esercizi sui numeri complessi volume 2 (per matematica o fisica)

 
Approfondisci la tua comprensione dei numeri complessi con il nostro documento “Esercizi sui numeri complessi volume 2 (per matematica o fisica)”. Questo articolo è un tesoro di esercizi pratici, pensati per sviluppare la tua abilità nel maneggiare i numeri complessi in varie situazioni matematiche. Dai problemi algebrici ai sistemi complessi, passando per equazioni e disuguaglianze, ogni argomento è trattato con cura per garantire una comprensione profonda e applicativa. Se sei uno studente o un professionista nel campo della matematica, questo documento è un must per elevare le tue competenze e intuizioni matematiche!

Gli esercizi presenti in questo articolo spaziano vari argomenti del mondo dei numeri complessi, per i lettori interessati ad uno in particolare tra questi argomenti di seguito sono elencati una serie di articoli specifici contenenti esercizi mirati.


 
 

Autori e revisori degli esercizi sui numeri complessi


 

Un ripasso di teoria

Leggi...

In questa sezione iniziale richiamiamo brevemente alcuni risultati e nozioni già viste in dettaglio nel primo volume, in modo da rendere più chiari i vari passaggi delle soluzioni agli esercizi seguenti.

    \[\]

    \[\]

Rappresentazione cartesiana e trigonometrica

Sia z \in \mathbb C un numero complesso. La rappresentazione cartesiana, che si ottiene identificando \mathbb C con \mathbb R^2, è data da

    \[z = x + \imath y,\]

dove x = \mathfrak{Re}(z) e y = \mathfrak{Im}(z) sono due numeri reali. Il coniugio di z, denotato con \bar z, si può scrivere in forma cartesiana come

    \[z = x + \imath y \implies \bar z = x - \imath y,\]

da cui segue che

    \[\mathfrak{Re}(z)=\frac{z+\bar z}{2} \quad \text{e} \quad \mathfrak{Im}(z)=\frac{z-\bar z}{2 \imath}.\]

Il modulo di z è dato da

    \[|z| = \sqrt{ \mathfrak{Re}(z)^2 + \mathfrak{Im}(z)^2},\]

ed è facile vedere che |z| \ge 0 ed è uguale a zero se e solo se z = 0. Infine, la rappresentazione trigonometrica di z è data da

    \[z = |z| (\cos \vartheta + \imath \sin \vartheta),\]

dove \vartheta è l’argomento di z che ricordiamo essere definito come segue:

Definizione 1. Sia z \neq 0. L’argomento principale di z, denotato \text{Arg }z, è l’unica soluzione dell’equazione reale

    \[\cos \left(\text{Arg }z\right) = \frac{\mathfrak{Re}(z)}{|z|} \quad \text{con vincolo } \text{Arg }z \in [-\pi,\pi).\]

L’argomento di z è, invece, una funzione a più valori che si ottiene rimuovendo il vincolo nell’equazione precedente, ottenendo infinite soluzioni:

(1)   \begin{equation*}  \arg z = \left\{ \text{Arg } z + 2 \pi k \: : \: k \in \mathbb{Z} \right\}.\end{equation*}

 

    \[\]

    \[\]

Argomento di un numero complesso

Iniziamo questo breve sommario sull’argomento osservando che la (1) si può “invertire”, ottenendo una formula per l’argomento principale:

    \[\text{Arg }z = \vartheta + \left\lfloor \frac{1}{2} - \frac{\vartheta}{2 \pi} \right\rfloor \cdot 2 \pi, \quad \text{per ogni } \vartheta \in \arg z.\]

La funzione \lfloor \cdot \rfloor : \mathbb{R}\to \mathbb{Z} è quella che associa ad ogni numero reale il più grande intero minore o uguale, ovvero \lfloor x \rfloor è l’unico intero tale che

    \[x - 1 < \lfloor x \rfloor \le x.\]

Tra le proprietà più importanti da conoscere ricordiamo l’argomento del prodotto (e del rapporto) tra numeri complessi, ovvero vale

(2)   \begin{equation*} \begin{aligned} 		& \text{Arg }(z_1z_2) = \text{Arg }z_1 + \text{Arg }z_2 + 2 \pi N_+, 		\\ & \text{Arg }(z_1/z_2) = \text{Arg }z_1 - \text{Arg }z_2 + 2\pi N_-, \end{aligned} \end{equation*}

dove N_{\pm} sono fattori correttivi definiti come segue:

(3)   \begin{equation*}  	N_\pm(z_1,z_2) := \begin{cases} -1 & \text{se } \text{Arg } z_1 \pm \text{Arg }z_2 > \pi, \\  0& \text{se } -\pi<\text{Arg }z_1 \pm \text{Arg }z_2 \le \pi, \\ 1 & \text{se } \text{Arg }z_1 \pm \text{Arg } z_2 \le -\pi,\end{cases}  \end{equation*}

Esempio 1. Se prendiamo z_1 = 1 e z_2 = z si vede immediatamente che per ogni z \neq 0 il valore principale dell’argomento dell’inverso z^{-1} è dato da

    \[\text{Arg }(1/z) = \text{Arg }\bar{z} = \begin{cases} \text{Arg }z &\text{se } \mathfrak{Im}(z)=0, \\ - \text{Arg }z & \text{se } \mathfrak{Im}(z)\neq0. \end{cases}\]

    \[\]

Un’altra proprietà interessante del valore principale riguarda le potenze. Infatti, dato z \in \mathbb C diverso da zero si può verificare per induzione che

(4)   \begin{equation*} 	\text{Arg }z^n = n \text{Arg }z + 2 \pi N_n, \end{equation*}

dove N_n è un intero che dipende da n ed è definito come segue:

    \[N_n := \left\lfloor \frac{1}{2} - \frac{n}{2\pi} \text{Arg }z\right\rfloor.\]

Per quanto riguarda la funzione a più valori \arg z, invece, ci sono alcune differenze importanti. Ad esempio, si può verificare che

    \[\begin{aligned} 	& \arg (z_1z_2) = \arg z_1 + \arg z_2, 	\\ & \arg(z_1/z_2) = \arg z_1 - \arg z_2, 	\\ & \arg(1/z) = \arg \bar z = -\arg z, \end{aligned}\]

e queste sono tutte da intendersi come uguaglianze tra insiemi trattandosi di funzioni a più valori.

    \[\]

    \[\]

Teorema di Eulero e formula di De Moivre

Una funzione che dai reali si può estendere senza troppe difficoltà ai complessi è quella esponenziale, ovvero ponendo

    \[\mathrm{e}^z := \mathrm{e}^x \mathrm{e}^{\imath y}.\]

Il primo fattore \mathrm{e}^x non è altro che l’esponenziale di un numero reale, perciò è sufficiente dare un significato al secondo fattore.

Teorema 2. (Eulero) Sia y \in \mathbb{R}. Allora si ha

(5)   \begin{equation*} 					\mathrm{e}^{\imath y} = \cos y + \imath \sin y. 				\end{equation*}

 

Una conseguenza immediata della formula di Eulero è la rappresentazione polare di un numero complesso z, che è data da

    \[z = |z| \mathrm{e}^{\imath \vartheta} \quad \text{con } \vartheta \in \arg z,\]

e si ottiene sostituendo (5) nella rappresentazione trigonometrica di z. Da questa segue immediatamente una formula per calcolare la potenza n-esima:

Proposizione 3. (Formula di De Moivre) Sia z \in \mathbb{C} ed n \in \mathbb{Z}. Allora

(6)   \begin{equation*}  					z^n = \rho^n ( \cos n \vartheta + \imath \sin n \vartheta), 				\end{equation*}

dove \rho = |z| e \vartheta = \text{Arg }z.

 

    \[\]

    \[\]

Radici n-esime dei numeri complessi

Fissato w \in \mathbb{C} vogliamo trovare tutte le soluzioni dell’equazione z^n = w con n \ge 1. Se w = 0 è banale, quindi supponiamo w \neq 0 e scriviamo

    \[z = \rho (\cos \vartheta + \imath \sin \vartheta) \quad \text{e} \quad w = r (\cos \alpha + \imath\sin\alpha),\]

con \alpha \in [-\pi,\pi). Utilizzando la (6) l’equazione z^n = w si può riscrivere come

    \[\rho^n ( \cos n \vartheta + \imath \sin n \vartheta) = r (\cos \alpha + \imath \sin \alpha),\]

e questa è del tutto equivalente al sistema reale

    \[\begin{cases} \rho^n = r, \\ n \vartheta = \alpha + 2\pi k, & \vartheta \in [-\pi,\pi).  \end{cases}\]

La prima ha soluzione \rho = r^{1/n} (radice n-esima positiva), mentre la seconda ha esattamente n soluzioni distinte nell’intervallo [-\pi,\pi) date da

(7)   \begin{equation*}  	\vartheta_k = \frac{\alpha + 2\pi k}{n}, \quad k = -\ell_1,\ldots,\ell_2, \end{equation*}

dove \ell_1,\ell_2 \in \mathbb N sono scelti in modo tale che \ell_1+\ell_2+1=n e valgono

    \[\frac{\alpha - 2 \ell_1 \pi}{n} \ge - \pi \quad \text{e} \quad \frac{\alpha + 2 \ell_2 \pi}{n} < \pi.\]

In particolare, il valore di \ell_1 ed \ell_2 dipende da \alpha e assicurano che tutti i valori di \vartheta rimangano nell’intervallo dell’argomento principale, ovvero [-\pi,\pi).

Osservazione 4. L’intervallo [-\pi,\pi) per l’argomento principale è scelto in maniera arbitraria e, pertanto, se stiamo semplicemente risolvendo un’equazione può essere più comodo considerare

    \[\vartheta_k = \frac{\alpha + 2\pi k}{n}, \quad k =0,\ldots,n-1\]

per ottenere tutte le soluzioni. Queste potrebbero non essere tutte in [-\pi,\pi), ma sono necessariamente contenute in un intervallo di ampiezza 2 \pi.

    \[\]

Tornando al discorso precedente, abbiamo dimostrato che ci sono esattamente n radici n-esime distinte di w e queste sono date da

    \[z_k = r^{1/n} \left( \cos  \frac{\alpha + 2\pi k}{n} + \imath \sin  \frac{\alpha + 2\pi k}{n} \right), \quad k = -\ell_1,\ldots,\ell_2.\]

I numeri z_{-\ell_1},\ldots,z_{\ell_2} giacciono tutti sulla circonferenza di raggio |w|^{1/n} e ciascuno forma un angolo di 2\pi/n con il precedente, perciò sono i vertici dell’n-poligono inscritto nella circonferenza di cui sopra.


 

Testi degli esercizi

 
In questo documento vedremo alcuni esercizi misti (con soluzioni) sui numeri complessi per fare pratica con tutte le nozioni introdotte nel primo volume.

Per una raccolta esaustiva di esercizi standard sui numeri complessi, il lettore può consultare, ad esempio, il libro [1], mentre per esercizi di livello avanzato (alcuni dei quali li vedremo qui), invece, raccomandiamo l’eserciziario [2].
 

Esercizi algebrici e calcolo di radici quadrate/cubiche

Esercizio 1  (\bigstar\largewhitestar\largewhitestar\largewhitestar\largewhitestar). Scrivere in forma algebrica i seguenti numeri complessi:

 

  1. z_1 = \dfrac{3-\imath}{4-\imath}
  2.  

  3. z_2 = \dfrac{4-3\imath}{(2+\imath)^2}
  4.  

  5. z_3 = \dfrac{(2\sqrt{3}+\imath)^3}{\sqrt{3}-\imath}
  6.  

  7. z_4 = \dfrac{\mathrm{e}^{2+\imath}}{\mathrm{e}^{3-2\imath}}
  8.  

  9. z_5 = \dfrac{2-\imath}{2+\imath}
  10.  

  11. z_6= \mathrm{e}^{-2+3\imath}
  12.  

  13. z_7 = \mathrm{e}^{(2+\imath)^3}
  14.  

  15. z_8 = \mathrm{e}^{(1-\imath)^6}

Suggerimento.

Nel caso di un rapporto tra due numeri complessi si può “razionalizzare” il denominatore. Per quanto riguarda i numeri complessi in forma esponenziale, è utile ricordarsi la formula di Eulero:

    \[\mathrm{e}^{\alpha + \imath \beta} = \mathrm{e}^\alpha \left[ \cos \beta + \imath \sin \beta \right].\]

Introduzione.

Poiché il procedimento è pressoché identico per tutti i casi, ne illustreremo solo alcuni esempi tratti dall’esercizio e lasceremo gli altri al lettore come esercizio pratico. Consideriamo due scenari distinti:

Svolgimento punti 1 e 3.

Se z è dato dal rapporto tra due numeri complessi, ovvero

(8)   \begin{equation*} z = \frac{\alpha + \imath \beta}{\delta + \imath \gamma}, \end{equation*}

allora si può “razionalizzare” moltiplicando e dividendo per il coniugato del denominatore. In particolare, abbiamo l’uguaglianza

    \[z = \frac{\alpha + \imath \beta}{\delta + \imath \gamma} \cdot \frac{\delta - \imath \gamma}{\delta - \imath \gamma} = \frac{(\alpha + \imath \beta)(\delta - \imath \gamma) }{\delta^2 + \gamma^2}.\]

A questo punto, solo il numeratore risulta essere un numero complesso ed è facile identificare parte reale e immaginaria di z, risolvendo così l’esercizio:

    \[z = \frac{1}{\delta^2+\gamma^2} \left[ (\alpha \delta + \beta \gamma) + \imath ( \beta \delta - \alpha \gamma )\right].\]

Applichiamo ora quanto detto all’esercizio. Ad esempio, dato z_1 si vede immediatamente che

    \[z_1 = \frac{3-\imath}{4-\imath} \cdot \frac{4+\imath}{4+\imath} = \frac{13 - \imath}{17} = \frac{13}{17} - \frac{1	}{17}\imath.\]

Se consideriamo z_3, per ridursi alla forma (8) è prima necessario sviluppare il numeratore come segue:

    \[\begin{aligned} (2\sqrt{3}+\imath)^3 & = 8 \cdot 3 \sqrt 3 + \imath^3 + 6 \sqrt3 \imath^2 + 3 (2 \sqrt3)^2 \imath 			\\ & = 24 \sqrt3 - \imath - 6 \sqrt3 + 36 \imath 			\\ & = 18 \sqrt{3} + 35 \imath. \end{aligned}\]

A questo punto si procede come nel caso precedente per concludere:

    \[z_3 = \frac{18 \sqrt{3} + 35 \imath}{ \sqrt{3}-\imath} \cdot \frac{\sqrt{3}+\imath}{\sqrt{3}+\imath}= \frac{54+18\sqrt{3}\imath+35\sqrt{3}\imath-35}{4} =  \frac{19}{4} + \frac{53 \sqrt{3}}{4} \imath.\]

Svolgimento punti 4, 6 e 8.

Se z è dato in forma esponenziale, ovvero z = \mathrm{e}^w per qualche w \in \mathbb C, allora è sufficiente scrivere w = \alpha + \imath \beta e poi applicare la formula di Eulero:

    \[z = \mathrm{e}^{\alpha + \imath \beta} = \mathrm{e}^\alpha \left[ \cos \beta + \imath \sin \beta\right].\]

Ad esempio, abbiamo

    \[z_6 = \mathrm{e}^{-2} \mathrm{e}^{3 \imath} = \frac{ \cos (3)}{\mathrm{e}^2} +  \imath \frac{\sin(3)}{\mathrm{e}^2},\]

o, analogamente, anche

    \[z_4 = \mathrm{e}^{2+\imath - (3-2\imath)} =  \mathrm{e}^{-1+3\imath} = \frac{\cos(3)}{\mathrm{e}} + \imath \frac{\sin(3)}{\mathrm{e}} .\]

Il caso di z_8 è leggermente diverso da trattare perché, prima di applicare la formula data sopra, bisogna calcolare esplicitamente l’esponente

    \[(1- \imath)^6.\]

In questi casi, piuttosto che sviluppare il prodotto esplicitamente, conviene scrivere il numero complesso in forma trigonometrica/esponenziale, ovvero

    \[1 - \imath = \sqrt{2} \mathrm{e}^{- \frac\pi4 \imath},\]

e poi prenderne la potenza richiesta:

    \[(1-\imath)^6 = 8 \mathrm{e}^{- \frac32 \pi \imath + 2 \pi \imath} = 8 \mathrm{e}^{\frac\pi2 \imath} = 8 \imath.\]

Abbiamo aggiunto 2 \pi perché, per convenzione, l’argomento principale è quello contenuto nell’intervallo [-\pi,\pi). A questo punto è facile vedere che

    \[z_8 = \mathrm{e}^{(1-\imath)^6} = \mathrm{e}^{8 \imath} = \cos 8 + \imath \sin 8.\]


 

Esercizio 2  (\bigstar\largewhitestar\largewhitestar\largewhitestar\largewhitestar). Calcolare

    \[z = \sqrt{ \frac{(1+\imath)^2}{(1-\imath)^3} }.\]

Suggerimento.

Scrivere il numero complesso sotto radice in forma algebrica w = a + \imath b e poi risolvere l’equazione

    \[z^2 = w\]

per trovare le due radici quadrate di w.

Svolgimento.

Iniziamo con lo scrivere il numero complesso dentro la radice in forma algebrica. Sviluppando le potenze si ottiene

    \[w := \frac{(1+\imath)^2}{(1-\imath)^3} = \frac{1 - 1 + 2 \imath}{1 + \imath - 3 \imath - 3} = \frac{\imath}{-1-\imath},\]

per cui possiamo razionalizzare come fatto nell’esercizio precedente:

    \[w = \frac{\imath}{-1-\imath} \cdot \frac{-1+\imath}{-1+\imath} = - \frac12 (1+\imath).\]

Di conseguenza, per concludere l’esercizio ci basta trovare le due soluzioni dell’equazione complessa

    \[z^2 = w.\]

Per risolverla possiamo, ad esempio, sfruttare la forma trigonometrica di un numero complesso. Infatti, se scriviamo

    \[w = \rho \left[ \cos \vartheta + \imath \sin \vartheta \right],\]

allora un semplice calcolo mostra che

    \[\rho = \sqrt{ \left(-\frac12\right)^2+\left(-\frac12\right)^2 } = \frac{\sqrt 2}{2} \quad \text{e} \quad \vartheta = -\frac34 \pi.\]

L’equazione si può dunque riscrivere come

    \[z^2 = \frac{\sqrt 2}{2} \left[ \cos \left( -\frac34 \pi \right) + \imath \sin \left( -\frac34 \pi \right) \right],\]

da cui segue immediatamente che

    \[z_1 =\frac{1}{\sqrt[4]{2}}  \left[ \cos \frac58 \pi + \imath \sin \frac58 \pi \right] \quad \text{e} \quad z_2 = \frac{1}{\sqrt[4]{2}} \left[ \cos \left( -\frac38 \pi \right) + \imath \sin \left( -\frac38 \pi \right) \right]\]

sono le due soluzioni dell’equazione.


 

Esercizio 3  (\bigstar\largewhitestar\largewhitestar\largewhitestar\largewhitestar). Calcolare

    \[z =\sqrt{ \frac{\sqrt{3}}{3}+\imath }.\]

Suggerimento.

Il numero w sotto radice è già scritto in forma algebrica, quindi è sufficiente risolvere l’equazione

    \[z^2 = w.\]

Svolgimento.

Il numero sotto radice è già scritto in forma a + \imath b, quindi l’esercizio è del tutto equivalente a risolvere l’equazione complessa

    \[z^2 = \frac{\sqrt{3}}{3} + \imath =:w.\]

Possiamo scrivere w in forma trigonometrica osservando che questo ha modulo e argomento principale rispettivamente dati da

    \[|w| = \sqrt{ \frac39 + 1} = \frac{2\sqrt3}{3} \quad \text{e} \quad \text{Arg } w = \frac\pi3.\]

Di conseguenza, l’equazione è equivalente a

    \[z^2 = \frac{2\sqrt3}{3} \left[ \cos \frac\pi3 + \imath \sin \frac\pi3 \right],\]

da cui si ottiene immediatamente che le due radici sono

    \[z_1= \frac{\sqrt{2}}{\sqrt[4]{3}} \left[\cos \frac\pi6 + \imath \sin \frac\pi6 \right]  \quad \text{e} \quad  z_2= \frac{\sqrt{2}}{\sqrt[4]{3}} \left[\cos \left(- \frac56 \pi \right) + \imath \sin \left(- \frac56 \pi \right) \right],\]

concludendo così l’esercizio.


 

Esercizio 4  (\bigstar\largewhitestar\largewhitestar\largewhitestar\largewhitestar). Calcolare

    \[\sqrt{1+\sqrt{3}\imath}.\]

Suggerimento.

Come l’esercizio precedente.

Svolgimento.

Calcolare le radici quadrate di w := 1 + \sqrt 3 \imath è equivalente a trovare le soluzioni dell’equazione complessa

    \[z^2 = w.\]

Mostriamo come sfruttare la forma esponenziale per risolverla. È facile vedere che

    \[|w| = \sqrt{1 + 3} = 2 \quad \text{e} \quad \text{Arg } w = \arctan \sqrt{3} = \frac\pi3,\]

per cui l’equazione sopra è del tutto equivalente al sistema reale

    \[\begin{cases} |z|^2 = 2, \\ 2 \vartheta = \frac\pi3 + 2k \pi, \quad k \in \mathbb Z.\end{cases}\]

La prima equazione ha come unica soluzione |z|=\sqrt{2}, mentre per quanto riguarda l’argomento abbiamo infinite soluzioni

    \[\vartheta_k = \frac\pi6 + k \pi, \quad k \in \mathbb Z.\]

Di conseguenza, per quanto riguarda l’argomento principale (e quindi nell’intervallo [-\pi,\pi)) abbiamo soltanto due soluzioni, ovvero

    \[\vartheta_1 = \frac\pi6 \quad \text{e} \quad \vartheta_2 = \frac\pi6 - \pi = - \frac56 \pi.\]

In particolare, le due radici quadrate di w sono

    \[z_1 = \sqrt{2} \mathrm{e}^{\imath \frac\pi6} \quad \text{e} \quad 	z_2 = \sqrt{2} \mathrm{e}^{-\imath \frac56 \pi}.\]


 
 

Esercizio 5  (\bigstar\bigstar\largewhitestar\largewhitestar\largewhitestar). Calcolare le radici cubiche di w=2+2\imath.

Suggerimento.

In questo caso conviene scrivere w in forma esponenziale e poi risolvere

    \[(|z|\mathrm{e}^{\imath \text{Arg } z})^3 = w \mathrm{e}^{\imath \text{Arg } w}\]

tramite il sistema equivalente di equazioni reali.

Svolgimento.

Iniziamo con lo scrivere w in forma esponenziale. Un semplice calcolo ci mostra che il suo modulo è dato da

    \[|w| = \sqrt{2^2 + 2^2} = \sqrt{8} = 2 \sqrt{2},\]

mentre il suo argomento principale è

    \[\text{Arg }w = \arctan \frac22 = \arctan 1 = \frac\pi4,\]

perciò si ha w = 2 \sqrt{2} \mathrm{e}^{\imath \frac\pi4}. A questo punto non ci rimane altro da fare che risolvere l’equazione complessa

    \[z^3 = 2 \sqrt{2} \mathrm{e}^{\imath \frac\pi4},\]

che, scrivendo anche z in forma esponenziale come |z|\mathrm{e}^{\imath \text{Arg } z}, sappiamo essere del tutto equivalente al sistema

    \[\begin{cases} |z|^3 = 2 \sqrt{2}, \\ 3 \text{Arg }z = \frac\pi4 + 2k \pi, \quad \text{Arg } z \in [-\pi,\pi). \end{cases}\]

La prima equazione ha come unica soluzione |z|= \sqrt[6]{8} = \sqrt2, mentre la seconda ammette tre soluzioni, ovvero

    \[\vartheta_1 = \frac{\pi}{12}, \quad \vartheta_2 = \frac{9}{12}\pi = \frac34 \pi, \quad \vartheta_3 = - \frac{7}{12} \pi,\]

da cui si ha che le tre radici cubiche di w sono

    \[z_1 = \sqrt2 \mathrm{e}^{\imath \frac{\pi}{12}}, \quad z_2 = \sqrt2 \mathrm{e}^{\imath \frac{3}{4}\pi}, \quad z_3 = \sqrt2 \mathrm{e}^{- \imath \frac{7}{12}\pi}.\]


 
 

Esercizio 6  (\bigstar\largewhitestar\largewhitestar\largewhitestar\largewhitestar). Determinare le radici quadrate e cubiche dei seguenti numeri complessi:

    \[\begin{aligned} &z_1 =-3, \quad z_2 = 1 - \sqrt{3}\imath, \quad z_3 = - 1 - 2 \imath, 					\\&z_4 = -\imath, \quad z_5 = 1-\imath, \quad z_6 = 2 + \imath. 				\end{aligned}\]

Suggerimento.

Trovare le radici quadrate e cubiche di un numero complesso w corrisponde, rispettivamente, a risolvere le equazioni

    \[z^2 = w \quad \text{e} \quad z^3 = w.\]

Svolgimento punto 1.

Dato che si tratta di ragionare allo stesso modo per ogni z_i, svolgiamo l’esercizio soltanto per due casi e lasciami gli altri al lettore.

Per calcolare le radici quadrate di z_1 dobbiamo risolvere

    \[z^2 = - 3,\]

ma questa è immediato vedere che ha soluzioni \pm \imath \sqrt{3}. Per le radici cubiche, invece, scriviamo z_1 in forma esponenziale,

    \[z_1 = 3 \mathrm{e}^{\imath \pi},\]

e consideriamo il sistema di equazioni reali associato a z^3 = z_1, ovvero

    \[\begin{cases} |z|^3 = 3, \\ 3 \text{Arg }z = \pi + 2k \pi & \text{Arg }z\in [-\pi,\pi). \end{cases}\]

La prima equazione ha come unica soluzione |z|=\sqrt[3]{3}, mentre la seconda ammette tre soluzioni distinte ovvero

    \[\vartheta_1 = \frac\pi3, \quad \vartheta_2 = - \frac\pi3, \quad \vartheta_3 = - \pi.\]

In particolare, le tre radici cubiche di z_1 sono

    \[z_{1,1} =\sqrt[3]{3} \mathrm{e}^{\imath \frac\pi3}, \quad  z_{1,2} =\sqrt[3]{3} \mathrm{e}^{-\imath \frac\pi3}, \quad  z_{1,3} =\sqrt[3]{3} \mathrm{e}^{-\imath \pi} = - \sqrt[3]{3}.\]

Svolgimento punto 6.

Per quanto riguarda z_6 = 2 + \imath conviene scriverlo in forma esponenziale anche per il calcolo delle radici quadrate. Si ha

    \[|z_6| = \sqrt{4 + 1} = \sqrt 5 \quad \text{e} \quad \vartheta:=\text{Arg }z_6 = \arctan \frac12 \approx 0.46,\]

avendo preso come argomento principale l’unica soluzione dell’arcotangente nell’intervallo [-\pi,\pi). Le radici quadrate sono le soluzioni di

    \[z^2 = \sqrt 5 \mathrm{e}^{\imath \vartheta},\]

e sono dunque date da

    \[(z_6^q)_{1,2} = \pm \sqrt[4]{5} \left[ \cos \left( \frac12 \arctan \frac12 \right)+\imath\sin \left( \frac12 \arctan \frac12 \right) \right].\]

Per quanto riguarda le radici cubiche, invece, dobbiamo risolvere

    \[z^3 = \sqrt 5 \mathrm{e}^{\imath  \vartheta},\]

che, come abbiamo visto in precedenza, è del tutto equivalente al seguente sistema di equazioni reali:

    \[\begin{cases} |z|^3 = \sqrt5, \\ 3\text{Arg }z = \vartheta + 2k \pi, & \text{Arg }z \in [-\pi,\pi). \end{cases}\]

La prima equazione ha come unica soluzione \sqrt[6]{5}, mentre la seconda ammette tre soluzioni distinte ovvero

    \[\text{Arg }(z_6^c)_1 = \frac\vartheta3, \quad  \text{Arg }(z_6^c)_2 = \frac\vartheta3 + \frac23 \pi ,\quad  \text{Arg }(z_6^c)_3 = \frac\vartheta3 - \frac23 \pi.\]

Di conseguenza, le tre radici cubiche sono date da

    \[\begin{aligned} & (z_6^c)_1 = \sqrt[6]{5} \left[ \cos \frac\vartheta3 + \imath \sin \frac\vartheta3 \right], \\ & (z_6^c)_{2,3}=\sqrt[6]{5} \left[ \cos \left( \frac\vartheta3 \pm \frac23 \pi\right) + \imath \sin \left( \frac\vartheta3 \pm \frac23 \pi\right) \right], \end{aligned}\]

concludendo l’esercizio. Osserviamo che è possibile semplificare (z_6^c)_{2,3} sfruttando le formule additive di seno e coseno, ovvero

    \[\begin{aligned} &  \sin(a+b)= \sin a \cos b + \sin b \cos a, \\ & \cos(a+b)= \cos a \cos b - \sin a \sin b, \end{aligned}\]

e poi il fatto che 2/3 \pi è un angolo notevole:

    \[\cos \frac23 \pi = - \frac12 \quad \text{e} \quad \sin \frac23 \pi = \frac{\sqrt 3}{2}.\]


 
 

Esercizio 7  (\bigstar\largewhitestar\largewhitestar\largewhitestar\largewhitestar). Calcolare le potenze dei seguenti numeri complessi:

 

  1. Se z = \imath, calcolare z^7 e z^{2002}.
  2.  

  3. Se z = 1+\imath, calcolare (z^{2005}+\bar{z}^{2005})/2^{1002}.
  4.  

  5. Se z = \frac12 - \imath \frac{\sqrt{3}}{2}, calcolare z^{8!-1}.

Suggerimento.

Utilizzare la forma trigonometrica o la forma esponenziale di un numero complesso.

Svolgimento punto 1.

Per il primo punto osserviamo che

    \[\imath^2=-1 \implies \imath^3 = - \imath \quad \text{e} \quad \imath^4 = 1.\]

Di conseguenza, per calcolare \imath^\alpha è sufficiente conoscere \alpha modulo 4 (ovvero, il resto della divisione per 4); più precisamente, si ha

    \[\imath^\alpha = \begin{cases} 1 & \text{se } \alpha \text{ è divisibile per } 4, \\ \imath & \text{se } \alpha \text{ diviso } 4 \text{ ha resto } 1, \\ -1 & \text{se } \alpha \text{ diviso } 4 \text{ ha resto } 2,\\ - \imath & \text{se } \alpha \text{ diviso } 4 \text{ ha resto } 3. \end{cases}\]

Svolgimento punto 2.

Per (2) scrivo z e \bar z in forma esponenziale, ovvero

    \[z = 1+\imath = \sqrt{2} \mathrm{e}^{\imath \frac\pi4} \implies \bar{z}=\sqrt{2} \mathrm{e}^{-\imath \frac\pi4},\]

da cui segue immediatamente che

    \[\frac{z^{2005}+\bar{z}^{2005}}{2^{1002}} = \frac{2^{1002+\frac12}}{2^{1002}} \left( \mathrm{e}^{ \imath 2005 \frac\pi4}+\mathrm{e}^{- \imath 2005 \frac\pi4} \right).\]

A questo punto osserviamo che

    \[2005 \cdot \frac\pi4 = 250 (2\pi) + \pi + \frac\pi4 \implies \mathrm{e}^{\imath 2005 \frac\pi4} = \mathrm{e}^{-\imath 3 \pi / 4},\]

ed analogamente

    \[-2005 \cdot \frac\pi4 = -250 (2\pi) - \pi - \frac\pi4 \implies \mathrm{e}^{-\imath 2005 \frac\pi4} =  \mathrm{e}^{\imath 3 \pi / 4}.\]

Sostituendo tutto nell’espressione precedente si trova

    \[\frac{z^{2005}+\bar{z}^{2005}}{2^{1002}} = \sqrt{2} \left(\mathrm{e}^{\imath \frac34 \pi}+\mathrm{e}^{-\imath \frac34 \pi}\right)= \sqrt{2} \cdot (-\sqrt{2}) = -2,\]

e questo conclude il secondo punto. Notiamo che la seconda uguaglianza è dovuta al fatto che, utilizzando la formula di Eulero

    \[ \mathrm{e}^{\imath \vartheta} = \cos \vartheta + \imath \sin \vartheta, \]

si ha

    \[\begin{aligned}\mathrm{e}^{\imath \frac34 \pi}+\mathrm{e}^{-\imath \frac34 \pi} &= \cos \frac34 \pi + \cos \left(-\frac34 \pi\right) + \imath \left[ \sin \frac34 \pi + \sin \left(-\frac34 \pi\right) \right]  	\\ & = 2 \cos \frac34 \pi + \imath \left[ \sin \frac34 \pi - \sin \frac34 \pi \right] 	\\ & = 2 \cos \frac34\pi = 2 \left(- \frac{\sqrt 2}{2} \right) = -\sqrt2.  \end{aligned}\]

Svolgimento punto 3.

Anche per risolvere (3) conviene scrivere z in forma esponenziale, ovvero

    \[z = \frac12 - \imath \frac{\sqrt 3}{2} = \mathrm{e}^{-\imath \frac\pi3}.\]

Poiché questo ha modulo unitario, bisogna soltanto capire come la potenza agisce sull’argomento principale. In particolare, dato che

    \[- \frac\pi3(8!-1) = \left[- 13440 + \frac13 \right]\pi =  2\pi \cdot (-6720) + \frac13 \pi,\]

sostituendo e ricordando che \mathrm{e}^{2\pi \imath}=1, si trova che

    \[z^{8!-1} = \mathrm{e}^{2\pi\imath \cdot (-6720)}  \mathrm{e}^{\imath \frac\pi3} = \mathrm{e}^{\imath \frac\pi3} = \frac12 + \imath \frac{\sqrt{3}}{2},\]

e questo conclude l’esercizio.


 
 

Esercizio 8  (\bigstar\bigstar\largewhitestar\largewhitestar\largewhitestar). Dato \lambda \in \mathbb{R}, consideriamo il numero complesso

    \[z = \frac{\lambda + 4\imath}{1 + \lambda \imath}.\]

Stabilire per quali valori di \lambda si ha z reale e per quali z puramente immaginario.

Suggerimento.

Calcolare la parte reale e la parte immaginaria di z in funzione di \lambda e poi porle uguali a zero.

Svolgimento.

L’idea è quella di trovare parte reale e immaginaria di z e successivamente dedurre i valori di \lambda (se esistono) per cui si annullano. Si ha

    \[z =  \frac{\lambda + 4\imath}{1 + \lambda \imath} =  \frac{\lambda + 4\imath}{1 + \lambda \imath}\cdot \frac{1-\lambda \imath}{1 - \lambda \imath}= \frac{1}{1+\lambda^2} \left( \lambda - \lambda^2 \imath + 4 \imath + 4\lambda \right),\]

da cui ricaviamo immediatamente che

    \[\mathfrak{Re}(z) = \frac{1}{1+\lambda^2} \left( \lambda + 4 \lambda\right) = \frac{5 \lambda}{1+\lambda^2} \quad \text{e} \quad \mathfrak{Im}(z) = \frac{1}{1+\lambda^2} \left( 4 - \lambda^2 \right) \imath.\]

Questo significa che z è puramente immaginario se e solo se \lambda = 0, mentre z è un numero reale se e solo se \lambda = \pm 2.


 
 

Esercizio 9  (\bigstar\largewhitestar\largewhitestar\largewhitestar\largewhitestar). Siano z,w \in \mathbb{C} due numeri complessi che soddisfano le seguenti identità:

    \[z = -3 + 4 \imath \quad \text{e} \quad zw = - 14 + 2 \imath.\]

Trovare modulo e argomento (principale) di w.

Suggerimento.

Modulo e argomento principale di un prodotto sono dati da

    \[|zw|=|z||w| \quad\text{e}\quad \text{Arg } (zw)=\text{Arg } z + \text{Arg } w + 2\pi N_+,\]

dove N_+ è definito in (3).

Svolgimento.

Sfruttando la proprietà del modulo di un prodotto si trova immediatamente che

    \[|zw|=|z||w| \implies |w| = \frac{|zw|}{|z|} =  \frac{|-14 + 2\imath|}{|-3 + 4 \imath|} = \frac{\sqrt{14^2 + 2^2}}{\sqrt{3^2 + 4^2}} =  2 \sqrt{2},\]

mentre, per quanto riguarda l’argomento, la strada più semplice è quella di scrivere w in forma algebrica. In particolare, si ha

    \[(-3 + 4 \imath)w = - 14 + 2 \imath \implies (-3+4 \imath)(x+\imath y)=-14 + 2 \imath,\]

da cui si ottiene (eguagliando parte reale e immaginaria rispettivamente) il seguente sistema nelle variabili reali x ed y:

    \[\begin{cases} -3x - 4y = - 14, \\ -3y + 4x = 2.  \end{cases}\]

Si vede immediatamente che l’unica soluzione è x = y = 2, per cui w = 2 + 2\imath ha argomento principale

    \[\text{Arg }w = \arctan \frac{2}{2} = \arctan 1 = \frac{\pi}{4},\]

e questo conclude l’esercizio. Alternativamente, possiamo sfruttare la formula

(9)   \begin{equation*} \text{Arg }w=\text{Arg }(zw) - \text{Arg }z + 2\pi N_-, \end{equation*}

dove N_- è dato da (3), per ricavare l’argomento principale di w. Infatti, è facile vedere che

    \[\text{Arg }z =  \pi - \arctan \frac43 \quad \text{e} \quad \text{Arg }(zw) = \pi - \arctan \frac17,\]

per cui sostituendo in (9) si trova

    \[\text{Arg }w = - \arctan\frac17 + \arctan\frac43 + 2 \pi N_-.\]

Per arrivare allo stesso risultato ottenuto sopra bisogna, in qualche modo, gestire la somma di due arcotangenti. Per farlo, si può utilizzare la formula

(10)   \begin{equation*} \arctan x = \frac12 \imath \log \frac{1 - \imath x}{1+\imath x}, \end{equation*}

che esula dagli scopi di questo documento e quindi diamo per scontata giusto per far vedere come concludere l’esercizio. Si ha

    \[\begin{aligned}\arctan\frac43 - \arctan\frac17 &= \frac12 \imath \log \frac{1 - \imath \frac43}{1+\imath \frac43} - \frac12 \imath \log \frac{1 - \imath \frac17}{1+\imath \frac17} 		\\ & = \frac12 \imath \log \frac{1 - \imath \frac43}{1+\imath \frac43}  \frac{1 + \imath \frac17}{1-\imath \frac17} 		\\ & = \frac12 \imath \log \frac{1-\imath}{1+\imath} = \arctan 1, 	\end{aligned}\]

dove l’ultimo passaggio segue da (10). Allora

    \[\text{Arg }g w = \arctan 1 + 2\pi N_- = \frac\pi4,\]

e questo conclude.


 
 

Esercizio 10  (\bigstar\bigstar\largewhitestar\largewhitestar\largewhitestar). Mostrare che se z \in \mathbb{C} ha modulo unitario, allora

(11)   \begin{equation*} 					\mathfrak{Re} \left[ \frac{2z}{1-z^2} \right] = 0. 				\end{equation*}

Suggerimento.

Si può scrivere z in forma trigonometrica sfruttando il fatto che, avendo modulo unitario, esiste \vartheta 	\in [-\pi,\pi) tale che

    \[z = \cos \vartheta + \imath \sin \vartheta.\]

Svolgimento.

Poiché z ha modulo unitario, possiamo scriverlo in forma trigonometrica come z = \cos \vartheta + \imath \sin \vartheta. Si vede facilmente che

    \[\frac{1}{1-z} = \frac{1}{(1-\cos \vartheta) - \imath \sin \vartheta} ,\]

perciò “razionalizzando” il denominatore si ottiene

    \[\frac{1}{1-z} = \frac{(1-\cos \vartheta) + \imath \sin \vartheta}{(1-\cos \vartheta)^2 + \sin^2 \vartheta} = \frac{1}{2 - 2 \cos \vartheta} \left[ (1-\cos \vartheta) + \imath \sin \vartheta \right].\]

Analogamente si ha

    \[\frac{1}{1+z} = \frac{(1+\cos \vartheta) - \imath \sin \vartheta}{(1+\cos \vartheta)^2 + \sin^2 \vartheta} = \frac{1}{2 + 2 \cos \vartheta} \left[ (1+\cos \vartheta) - \imath \sin \vartheta \right],\]

per cui possiamo esprimere il denominatore del numero complesso in (11) moltiplicando le due espressioni appena trovate, ottenendo

    \[\frac{1}{1-z^2} = \frac{1}{4 - 4 \cos^2 \vartheta} \left[(1-\cos^2 \vartheta + \sin^2 \vartheta)  + \imath (2 \sin \vartheta \cos \vartheta)  \right].\]

Possiamo semplificare l’espressione sfruttando le identità trigonometriche

    \[\cos^2 \vartheta+\sin^2\vartheta = 1 \quad \text{e} \quad 2 \sin \vartheta \cos \vartheta = \sin 2\vartheta,\]

arrivando così all’uguaglianza

    \[\frac{1}{1-z^2} = \frac{1}{4 \sin^2 \vartheta} \left[ 2 \sin^2 \vartheta + \imath \sin 2 \vartheta \right].\]

Infine, si moltiplica tutto per z

    \[\frac{z}{1-z^2} = \frac{1}{4 \sin^2 \vartheta} \left[( 2 \sin^2 \vartheta \cos \vartheta - \sin \vartheta \sin 2 \vartheta ) + \imath ( 2 \sin^3 \vartheta + \cos \vartheta \sin 2 \vartheta) \right],\]

perciò la parte reale è data da

    \[\mathfrak{Re}\left[ \frac{z}{1-z^2} \right] = 2 \sin^2 \vartheta \cos \vartheta - \sin \vartheta \sin 2 \vartheta.\]

A questo punto, applicando di nuovo la formula di duplicazione del seno, si ottiene

    \[\mathfrak{Re} \left[ \frac{z}{1-z^2} \right]= 2 \sin^2 \vartheta \cos \vartheta - 2 \sin^2 \vartheta \cos \vartheta = 0,\]

e questo conclude l’esercizio perché abbiamo dimostrato che per |z|=1 la proprietà (11) è verificata.


 
 

Esercizio 11  (\bigstar\bigstar\bigstar\largewhitestar\largewhitestar). Siano z,w \in \mathbb{C} due numeri complessi che soddisfano \bar{z}w \neq 1 e tali che |z|=1 oppure |w|=1. Si mostri che

    \[\left| \frac{z-w}{1-\bar{z}w} \right|=1.\]

Suggerimento.

Per semplificare il rapporto nel modulo si può moltiplicare e dividere per il coniugato del denominatore,

    \[\frac{z-w}{1-\bar z w} \frac{1-z\bar w}{1 - z \bar w} = \frac{z-w - z^2\bar w + z |w|^2}{1-|z|^2|w|^2},\]

in modo tale che la proprietà da dimostrare sia del tutto equivalente a

    \[|z-w-z^2\bar w + z|w|^2| = 1-|z|^2|w|^2.\]

Svolgimento.

Si può procedere come nel suggerimento oppure ragionare in maniera leggermente diversa. Ad esempio, sapendo che vale

    \[1 = \left| \frac{z-w}{1-\bar z w} \right|= \frac{|z-w|}{|1-\bar z w|},\]

e 1 - \bar z w \neq 0 per ipotesi, è del tutto equivalente far vedere che vale l’uguaglianza

    \[|z-w|=|1-\bar{z}w|.\]

Inoltre, trattandosi di due termini non negativi sotto radice, possiamo prendere il quadrato di entrambi ed ottenere l’equazione

    \[ |z-w|^2 = |1-\bar z w|^2. \]

Supponiamo |z|=1 (il caso |w|=1 si fa nello stesso modo) e scriviamo i due numeri complessi in forma trigonometrica come segue:

    \[z = \cos \vartheta + \imath \sin \vartheta \quad \text{e} \quad w = |w| \left( \cos \beta + \imath \sin \beta \right).\]

A questo punto, il modulo della differenza al quadrato è dato da

    \[\begin{aligned} |z-w|^2 & = (\cos \vartheta - |w| \cos \beta)^2 + (\sin \vartheta -|w| \sin \beta)^2  	\\ & = \cos^2 \vartheta + |w| \cos^2 \beta - 2 |w| \cos \vartheta \cos \beta + \sin^2 \vartheta + |w|^2 \sin^2 \beta - 2|w| \sin \vartheta \sin \beta 	\\ & = \left( \cos^2 \vartheta + \sin^2 \vartheta \right) + |w|^2 \left( \cos^2 \beta + \sin^2 \beta \right) - 2 |w| \left( \sin \vartheta\sin \beta + \cos \vartheta \cos \beta \right) 	\\ & = 1 + |w|^2 - 2 |w| \cos(\beta - \vartheta), \end{aligned}\]

dove nell’ultimo passaggio abbiamo utilizzato l’identità fondamentale trigonometrica e la formula di sottrazione del coseno, che ricordiamo essere

    \[\cos(b-a)=\cos(a-b) = \cos a \cos b + \sin a \sin b.\]

Per quanto riguarda il prodotto \bar{z} w si ha

    \[\bar{z}w =|w| \cos(\beta-\vartheta) + \imath |w|\sin(\beta-\vartheta),\]

da cui segue immediatamente che

    \[\begin{aligned} |1-\bar{z}w|^2 &= \left[ 1 - |w| \cos(\beta-\vartheta) \right]^2+ |w|^2\sin^2(\beta-\vartheta) 	\\ & = 1 + |w|^2 \left[ \cos^2 (\beta-\vartheta) + \sin^2 (\beta-\vartheta) \right] - 2 |w| \cos(\beta-\vartheta) 	\\ & = 1 + |w|^2 - 2 |w| \cos(\beta-\vartheta), \end{aligned}\]

e questo conclude.


 
 

Esercizio 12  (\bigstar\bigstar\largewhitestar\largewhitestar\largewhitestar). Si dimostri che

    \[\left| \frac{\mathrm{e}^{\imath xy} - 1}{y} \right| \le |x|\]

per ogni x,y \in \mathbb{R} con y \neq 0.

Suggerimento.

Si può utilizzare la formula di Eulero (5) oppure la definizione di esponenziale complesso.

Svolgimento.

Per la formula di Eulero (5) si ha

    \[\mathrm{e}^{\imath xy} = \cos xy + \imath \sin xy,\]

per cui possiamo riscrivere il termine a sinistra come segue:

    \[\frac{\mathrm{e}^{\imath xy} - 1}{y} = \frac{ (\cos xy - 1) + \imath \sin xy }{ y }.\]

Il modulo è dato da

    \[\begin{aligned} \left| \frac{\mathrm{e}^{\imath xy} - 1}{y} \right| & =\frac{1}{|y|}\sqrt{ \cos^2 xy + 1 - 2 \cos xy + \sin^2 xy }  	\\ & = \frac{1}{|y|} \sqrt{2 - 2 \cos xy}, \end{aligned}\]

per cui, utilizzando la formula di sdoppiamento del coseno,

    \[1 - \cos \alpha = \sin^2 \frac{\alpha}{2},\]

si arriva all’espressione seguente:

    \[\left| \frac{\mathrm{e}^{\imath xy} - 1}{y} \right| = \frac{1}{|y|}2 \left| \sin \frac{xy}{2} \right|.\]

Di conseguenza, per concludere l’esercizio è sufficiente verificare che

    \[\left| \sin \frac{xy}{2} \right| \le \frac{|x||y|}{2}\]

vale per ogni x,y \in \mathbb{R} con y \neq 0. A questo punto però ci ricordiamo di una disuguaglianza notevole1 secondo cui vale

    \[| \sin \alpha | \le |\alpha| \quad \text{per ogni } \alpha \in \mathbb{R}.\]

Applicandola al nostro caso si conclude dato che vale

    \[\left| \sin \frac{xy}{2} \right| \le \frac{|xy|}{2} = \frac{|x||y|}{2},\]

dove abbiamo sfruttato la proprietà del modulo |x||y|=|xy|.

 


    \[\]

  1. Per verificare questa disuguaglianza si può utilizzare, ad esempio, il teorema del valore intermedio nell’intervallo \alpha \in (0,1), mentre è banale per \alpha = 0 e \alpha \ge 1. Il caso \alpha < 0 segue immediatamente sfruttando la simmetria.

 
 

Esercizio 13  (\bigstar\bigstar\largewhitestar\largewhitestar\largewhitestar). Si dimostri che per ogni z \in \mathbb{C} \setminus \{1\} di modulo unitario esiste t \in \mathbb{R} tale per cui si può scrivere

    \[z = \frac{t+\imath}{t-\imath}.\]

Suggerimento.

Si razionalizza e poi si considera l’uguaglianza ottenuta

    \[(t+\imath)^2 = (t^2+1)z\]

come una equazione nella variabile reale t, sfruttando eventualmente la forma algebrica di z.

Svolgimento.

Il primo passo è quello di studiare il numero complesso

    \[\frac{t+\imath}{t-\imath}\]

al variare di t \in \mathbb{R}, esplicitando se possibile parte reale e immaginaria. Un semplice calcolo ci mostra che

    \[\frac{t+ \imath}{t- \imath} = \frac{(t+\imath)^2}{t^2 + 1} = \frac{1}{t^2+1} \left(t^2 - 1 + \imath 2t \right),\]

da cui segue immediatamente che

    \[\mathfrak{Re} \left[\frac{t+\imath}{t-\imath}\right] = \frac{t^2-1}{t^2+1}, \quad \text{e} \quad \mathfrak{Im}\left[\frac{t+\imath}{t-\imath}\right] = \frac{2 t}{t^2+1}.\]

Di conseguenza, se scriviamo z = u + \imath v dobbiamo far vedere che esiste almeno una soluzione reale t al seguente sistema:

    \[\begin{cases} t^2 - 1 = u(t^2+1), \\ 2t = v(t^2+1). \end{cases}\]

La seconda equazione ci da

    \[t^2 - \frac{2}{v} t + 1 = 0,\]

dove abbiamo diviso per v dato che per ipotesi z \neq 1 (quindi ha parte immaginaria non zero). Se ora andiamo a sostituire nella prima otteniamo

    \[\frac{2}{v}t - 2 = u \frac{2}{v}t \implies t = \frac{v}{1-u},\]

che è ben definito perché u \in [0,1) per ipotesi e questo conclude.


 
 

Esercizio 14  (\bigstar\bigstar\largewhitestar\largewhitestar\largewhitestar). Si dimostri che per ogni n \in \mathbb{N} si ha

    \[(-1+\imath \sqrt{3})^n + (-1 - \imath \sqrt{3})^n = \begin{cases} 2^{n+1} & \text{se $n$ è divisibile per $3$}, \\ - 2^n & \text{altrimenti.}\end{cases}\]

Suggerimento.

Utilizzare la forma trigonometrica ricordando che

    \[\left( \cos \vartheta + \imath \sin \vartheta \right)^n = \cos n \vartheta + \imath \sin n \vartheta.\]

Svolgimento.

In coordinate trigonometriche abbiamo

    \[- 1 \pm \imath \sqrt{3} = 2 \left[ \cos \left(\pm \frac{2}{3} \pi \right) + \imath \sin \left(\pm \frac{2}{3} \pi \right)\right],\]

dato che sono l’uno il coniugato dell’altro. Per (6) si ha

    \[(- 1 \pm \imath \sqrt{3})^n = 2^n \left[ \cos \left(\pm \frac{2}{3}n \pi \right) + \imath \sin \left(\pm \frac{2}{3}n \pi \right)\right],\]

per ogni n \in \mathbb{N}. Se n è divisibile per 3, diciamo n = 3k, allora

    \[(-1 \pm \imath \sqrt{3})^n = 2^n \left[ \cos \left( \pm 2k \pi \right) + \imath \sin \left( \pm 2k \pi \right) \right] =2^n,\]

da cui prendendo la somma si ottiene il risultato desiderato:

    \[(- 1 + \imath \sqrt{3})^n + (- 1 - \imath \sqrt{3})^n  =  2^n + 2^n = 2^{n+1}.\]

Se n non è divisibile per 3, sfruttando il fatto che \cos è pari e \sin è dispari otteniamo l’uguaglianza

    \[(- 1 + \imath \sqrt{3})^n + (- 1 - \imath \sqrt{3})^n = 2^{n+1} \cos \frac{2}{3} n \pi,\]

e questo conclude osservando che

    \[\cos\left( \frac{2}{3} \pi + 2k \pi \right) = -\frac12 \quad \text{e} \quad \cos \left( \frac{4}{3} \pi + 2k \pi \right) = - \frac12.\]


 
 

Esercizio 15  (\bigstar\bigstar\largewhitestar\largewhitestar\largewhitestar). Dato z \in \mathbb{C} qualsiasi, si dimostri che il prodotto della radici n-esime di z è uguale a (-1)^{n+1}z.

Suggerimento.

Scrivere le radici n-esime in forma polare e poi calcolare esplicitamente il prodotto.

Svolgimento.

Abbiamo visto che le radici n-esime di z non sono altro che le n soluzioni distinte dell’equazione z = w^n, ovvero

    \[w_k = |z|^{\frac1n} \mathrm{e}^{\imath \frac{\text{Arg }z + 2k \pi}{n}}, \quad k = 0, \ldots, n-1.\]

Notiamo che facendo variare k in questo modo c’è il rischio che da un certo punto in poi si arrivi ad avere

    \[\frac{ \text{Arg }z + 2k \pi }{n} \ge \pi,\]

ma in questo caso è meglio rinunciare alla convenzione di rimanere in [-\pi,\pi) a favore della chiarezza dato che si dovrebbe prendere

    \[w_k = |z|^{\frac1n} \mathrm{e}^{\imath \frac{\text{Arg }z + 2k \pi}{n}}, \quad k = -\ell_1, \ldots, \ell_2\]

con \ell_2+\ell_1= n-1 e in modo tale che

    \[- \pi \le \frac{ \text{Arg }z + 2k \pi }{n} < \pi \quad \text{per ogni } k \in \{-\ell_1,\ldots,\ell_2\}.\]

In ogni caso, il prodotto è dato da

    \[\prod_{k = 0}^{n-1} |z|^{\frac1n} \mathrm{e}^{ \imath \frac{\text{Arg }z + 2k \pi}{n}} = |z| \prod_{k = 0}^{n-1} \mathrm{e}^{\imath \frac{\text{Arg }z + 2k \pi}{n}},\]

e, sfruttando la proprietà della funzione esponenziale, la produttoria si trasforma in una sommatoria all’esponente:

    \[\prod_{k = 0}^{n-1} |z|^{\frac1n} \mathrm{e}^{ \imath \frac{\text{Arg }z + 2k \pi}{n}} = |z| \mathrm{e}^{\imath \sum_{k = 0}^{n-1} \frac{\text{Arg }z + 2k \pi}{n}}.\]

Ricordando la formula per la somma dei primi n-1 numeri interi si calcola esplicitamente l’esponente

    \[\begin{aligned} \sum_{k = 0}^{n-1} \frac{\text{Arg }z + 2k \pi}{n} & = n \frac{\text{Arg }z}{n} + \frac{2 \pi }{n} \sum_{k = 0}^{n-1}k  		\\ & = \text{Arg }z + \frac{2\pi}{n} \frac{(n-1)n}{2} 		\\ & = \text{Arg }z + (n-1) \pi, \end{aligned}\]

da cui segue che

    \[\prod_{k = 0}^{n-1} |z|^{\frac1n} \mathrm{e}^{\imath \frac{\text{Arg }z + 2k \pi}{n}} = \underbrace{|z| \mathrm{e}^{ \imath \text{Arg }z}}_{=z} \underbrace{\mathrm{e}^{\imath (n-1)\pi}}_{=(-1)^{n-1}} = (-1)^{n-1}z,\]

e questo conclude.


 
 

Equazioni e sistemi nei complessi

 

Esercizio 16  (\bigstar\largewhitestar\largewhitestar\largewhitestar\largewhitestar). Risolvere per x \in \mathbb R le seguenti equazioni nel campo dei numeri complessi:

 

  1. x^2-4x+5=0;
  2.  

  3. 2x^2-2x+1=0;
  4.  

  5. x^2-5x+7=0;
  6.  

  7. \imath x^2 -2x - 2 \imath = 0.

Suggerimento.

Si può applicare la formula risolutiva per equazioni di secondo grado facendo attenzione al segno del \Delta.

Svolgimento punto 1.

Ricordiamo che per una equazione generica del secondo ordine

(12)   \begin{equation*} x^2 + bx + c=0, \end{equation*}

la formula risolutiva è data da

    \[x_{1,2} = \frac{-b \pm \sqrt{\Delta} }{2}, \quad \text{con } \Delta = b^2 - 4c.\]

Per la prima equazione abbiamo \Delta = 16-20 = -4, per cui \sqrt{-4} non è definita nei numeri reali. Detto questo, sfruttiamo l’unità immaginaria per scrivere

    \[x_{1,2} = \frac{4 \pm \sqrt{-4}}{2} = 2 \pm \imath,\]

quindi le soluzioni sono complesse e coniugate1.

 


    \[\]

  1. Questo è un fatto valido in generale, ovvero ogni polinomio p(x) (di qualsiasi grado) a coefficienti reali soddisfa la seguente proprietà:

        \[p(z) = 0 \iff p(\bar z)=0.\]

Svolgimento punto 2.

Per la seconda dividiamo per riportarla alla forma (12), ottenendo

    \[2x^2-2x+1=0 \iff x^2 - x + \frac12 = 0.\]

Si ha

    \[x_{1,2} = \frac{1 \pm \sqrt{1 - 2}}{2} = \frac{1}{2} \pm  \frac{\imath}{2},\]

ovvero anche in questo caso le soluzioni sono complesse e coniugate.

Svolgimento punto 3.

Analogamente per la terza equazione abbiamo le due soluzioni

    \[x_{1,2} = \frac{5 \pm \sqrt{25-28}}{2} = \frac{5}{2} \pm \imath \frac{\sqrt{3}}{2}.\]

Svolgimento punto 4.

L’ultima equazione deve essere prima riportata nella forma (12), quindi dividiamo per \imath ed otteniamo

    \[\imath x^2 -2x - 2 \imath = 0 \iff x^2 - \frac2\imath x - 2 = 0.\]

Sfruttando l’identità \imath^2 = -1 il delta della equazione è dato da

    \[\Delta = \frac{4}{\imath^2} + (-4)(-2) = - 4 +8 = 4,\]

da cui le soluzioni della equazione sono

    \[x_{1,2} = \frac{\frac{2}{\imath} \pm 2}{2} = \frac1\imath \pm 1,\]

e questo conclude.

Osservazione.

È interessante notare che le soluzioni della quarta non sono coniugate. Il motivo è che l’equazione

    \[\imath x^2 - 2x - 2 \imath = 0\]

è a coefficienti complessi, ed è facile vedere che applicando il coniugio si ottiene una equazione non equivalente alla precedente, ovvero

    \[- \imath x^2 - 2x + 2 \imath = 0,\]

dato che non è possibile ottenerla dalla prima tramite nessuna manipolazione algebrica (ad esempio, moltiplicare per -1 o simili).


 
 

Esercizio 17  (\bigstar\largewhitestar\largewhitestar\largewhitestar\largewhitestar). Risolvere le seguenti equazioni nei complessi:

 

  1. | |z|-2\imath|^2 = 4;
  2.  

  3. |z|^2=12-|z|;
  4.  

  5. \mathfrak{Im}(z^2)=|z|^2;

Suggerimento.

Nella prima e nella terza conviene sviluppare il modulo e poi, se necessario, passare alla forma algebrica di z Nel secondo caso, invece,

    \[|z|^2+|z|-12 = 0\]

si può inizialmente trattare come una equazione nella variabile reale |z|.

Svolgimento punto 1.

Per la prima equazione osserviamo che |z| è un numero reale, perciò il modulo a sinistra si calcola facilmente come

    \[||z|-2\imath|^2 = |z|^2 + 4.\]

Ne segue immediatamente che

    \[|z|^2 + 4 = 4 \iff |z|^2 = 0,\]

e questa ha come unica soluzione z = 0 (per definizione di modulo).

Svolgimento punto 2.

La seconda si può risolvere considerando |z| come una variabile reale; si trova

    \[|z|^2 + |z| - 12 = 0 \iff |z| = \frac{-1 \pm 7}{2},\]

ma solo |z|=3 è ammissibile perché |z|\ge0. Dunque

    \[\{ z \in \mathbb{C} \: : \: |z|=3\}\]

è l’insieme delle soluzioni dell’equazione e coincide con il bordo della palla di centro l’origine e raggio 3.

Svolgimento punto 3.

Per la terza introduciamo la forma algebrica

    \[z = a + \imath b \implies z^2 = a - b + \imath(2ab),\]

da cui segue che

    \[2ab = \mathfrak{Im}(z^2)=|z|^2 = a^2+b^2.\]

Questa è ovviamente soddisfatta se e solo se a=b, ovvero la retta di coefficiente angolare uno e passante per l’origine

    \[\{ a(1+\imath) \in \mathbb{C} \: : \: a \in \mathbb{R} \}\]

è l’insieme delle soluzioni dell’equazione.


 
 

Esercizio 18  (\bigstar\largewhitestar\largewhitestar\largewhitestar\largewhitestar). Determinare per quali z \in \mathbb{C} si ha

    \[\imath z^2 + \mathfrak{Im}\left(z+\frac{17}{7}\right) = 21.\]

Suggerimento.

Sfruttare la forma algebrica di z.

Svolgimento.

Sia z = a + \imath b. Sostituendo nell’equazione si ottiene

    \[\imath(a^2 - b^2 + \imath 2ab) + b = 21,\]

e questa è equivalente (ponendo uguali a zero parte reale e immaginaria rispettivamente) al seguente sistema di equazioni reali:

    \[\begin{cases}-2ab + b = 21, \\ a^2-b^2=0.\end{cases}\]

La seconda equazione ci dice che a=b oppure a=-b.

 

  1. Sostituendo a = b nella prima equazione si ottiene

        \[-2a^2 + a = 21,\]

    ma questa ha discriminante dato da \Delta = 1 - 4 \cdot 42 < 0, quindi non ammette soluzioni reali.

  2.  

  3. Se a = -b, invece, si trova l’equazione

        \[2a^2-a = 21,\]

    ed è facile vedere con la formula risolutiva che questa ha due soluzioni reali e distinte, ovvero a_1 = -3 e a_2 = \frac72.

Di conseguenza l’equazione di partenza ha due soluzioni complesse, ovvero

    \[z_1 = -3 + 3 \imath \quad \text{e} \quad z_2=\frac72 - \frac72 \imath.\]


 
 

Esercizio 19  (\bigstar\largewhitestar\largewhitestar\largewhitestar\largewhitestar). Determinare per quali z \in \mathbb{C} si ha

    \[\frac{z^2}{4+|z^2|} + \frac{9}{13} = 0.\]

Suggerimento.

Sfruttare la forma algebrica di z.

Svolgimento.

Se z=a+\imath b allora il suo quadrato è dato da

    \[z^2 = (a^2-b^2) + \imath (2ab).\]

Quindi, tenendo conto che |z^2|=|z|^2, l’equazione si può riscrivere come

    \[\frac{a^2-b^2 + \imath 2ab}{4 + a^2+b^2} + \frac{9}{13}=0.\]

Di nuovo poniamo uguali a zero parte reale e immaginaria rispettivamente, ottenendo il seguente sistema di equazioni reali:

    \[\begin{cases} 13(a^2-b^2) = - 9(4+a^2+b^2) , \\ 2ab = 0.\end{cases}\]

Dalla seconda equazione otteniamo immediatamente a = 0 oppure b = 0. Nel primo caso, sostituendo si arriva a

    \[-13b^2 = - 36 - 9b^2 \implies 4b^2 = 36,\]

e questa ammette due soluzioni reali b = \pm 3. D’altra parte, se b = 0 l’equazione

    \[13a^2 = - 36 - 9a^2\]

non ammette soluzioni reali (discriminante negativo), perciò l’equazione iniziale ammette due soluzioni puramente immaginarie che sono date da

    \[z_1 = 3 \imath \quad \text{e} \quad z_2 = - 3 \imath.\]


 
 

Esercizio 20  (\bigstar\largewhitestar\largewhitestar\largewhitestar\largewhitestar). Determinare per quali z \in \mathbb{C} si ha

    \[z^2 + |z^2| + 2 \imath \bar{z}=0.\]

Suggerimento.

Sfruttare la forma algebrica di z.

Svolgimento.

Se z=a+\imath b abbiamo già mostrato che z^2 = (a^2-b^2) + \imath (2ab) per cui l’equazione si può riscrivere come

    \[a^2-b^2 + \imath 2ab + a^2+b^2 + 2 \imath(a-\imath b)=0.\]

Ponendo uguali a zero parte reale e immaginaria del termine a sinistra ci porta al seguente sistema di equazioni reali:

    \[\begin{cases} a^2+b=0,\\ ab +a = 0. \end{cases}\]

Dalla seconda equazione si ricava a = 0 oppure b = -1. Nel primo caso l’altra equazione ci dà b = 0 come unica soluzione, mentre nel secondo caso

    \[a^2-1 = 0 \iff a = \pm 1,\]

perciò l’equazione iniziale ha tre soluzioni complesse date da

    \[z_1=0, \quad z_2 = 1 - \imath, \quad z_1 = -1-\imath.\]


 
 

Esercizio 21  (\bigstar\largewhitestar\largewhitestar\largewhitestar\largewhitestar). Determinare per quali z \in \mathbb{C} si ha

    \[\frac{1+z^2}{|z^2|} = 10.\]

Suggerimento.

Come sopra.

Svolgimento.

Come fatto negli esercizi precedenti, se scriviamo z = a + \imath b l’equazione di partenza è del tutto equivalente a

    \[\frac{1+(a^2-b^2)+\imath 2ab}{a^2+b^2}=10,\]

e, portando tutto a sinistra, si arriva a

    \[\frac{1+(a^2-b^2) - 10(a^2+b^2) + \imath 2ab}{a^2+b^2}=0.\]

Poniamo parte reale e immaginaria uguali a zero rispettivamente per ottenere il seguente sistema di equazioni reali:

    \[\begin{cases} 1-9a^2-11b^2=0, \\ ab=0. \end{cases}\]

Dalla seconda equazione abbiamo a=0 oppure b = 0, ma a=b=0 non è ammissibile perché annulla il denominatore.

 

  1. Se a = 0 allora la prima equazione ci dà 1-11b^2=0, da cui si trovano le due soluzioni b=\pm 1/\sqrt{11}.
  2.  

  3. Se b = 0 allora 1-9a^2=0 ci dà le due soluzioni a = \pm 1/3.

In particolare, l’equazione iniziale ha quattro soluzioni, due reali e due puramente immaginarie, date da

    \[z_{1,2} = \pm \frac13 \quad \text{e} \quad  z_{3,4} = \pm \frac{1}{\sqrt{11}} \imath.\]


 
 

Esercizio 22  (\bigstar\largewhitestar\largewhitestar\largewhitestar\largewhitestar). Determinare per quali z \in \mathbb{C} si ha

    \[\frac{|z^4|}{z^2} = - 4.\]

Suggerimento.

Ricordiamo che il modulo soddisfa la seguente proprietà:

    \[|z|^2 = z \bar z \quad \text{per ogni } z \in \mathbb C.\]

Svolgimento.

Per risolvere questo esercizio è sufficiente sfruttare una proprietà del modulo, ovvero

    \[|z|^2 = z \bar{z} \implies |z^4|=|z|^4 = z^2 \bar{z}^2.\]

L’equazione si può riscrivere come

    \[\bar{z}^2 = -4,\]

e questa ha due soluzioni \bar{z}_{1,2}= \pm 2 \imath o, rispetto alla variabile z,

    \[z_{1,2}=\mp 2 \imath.\]


 
 

Esercizio 23  (\bigstar\largewhitestar\largewhitestar\largewhitestar\largewhitestar). Determinare per quali z \in \mathbb{C} si ha

    \[\frac{|z^5|}{z^2} = - 27.\]

Suggerimento.

Si può sfruttare il trucco dell’esercizio precedente e poi la forma esponenziale oppure trigonometrica.

Svolgimento.

Dalle proprietà del modulo si vede che

    \[|z|^2 = z \bar{z} \implies |z^5|=|z|^5 = |z||z|^4= |z| z^2 \bar{z}^2,\]

perciò l’equazione di partenza si può riscrivere come

    \[\bar{z}^2 |z| = -27.\]

Dato che -1 = \mathrm{e}^{\imath \pi}, per risolvere è conveniente passare alla forma esponenziale entrambi i membri dell’equazione, ottenendo

    \[|z|^3 \mathrm{e}^{-2\imath \text{Arg }z} = 27 \mathrm{e}^{\imath \pi},\]

che è equivalente al sistema di equazioni reali

    \[\begin{cases} |z|^3 = 27 \\ -2 \text{Arg }z = \pi + 2k \pi, & \text{Arg }z \in [-\pi,\pi). \end{cases}\]

La prima ha come unica soluzione |z| = 3 mentre la seconda ammette due soluzioni nell’intervallo ammissibile, ovvero

    \[\text{Arg }z = - \frac\pi2 \quad \text{e} \quad \text{Arg }z = \frac\pi2,\]

che corrispondono quindi alle due soluzioni complesse dell’equazione iniziale

    \[z_1 = 3 \mathrm{e}^{\imath \frac\pi2} = 3 \imath \quad \text{e} \quad z_2 = 3 \mathrm{e}^{-\imath \frac\pi2} = - 3 \imath.\]


 
 

Esercizio 24  (\bigstar\largewhitestar\largewhitestar\largewhitestar\largewhitestar). Determinare per quali z \in \mathbb{C} si ha

    \[z^2 = (\bar{z})^2(25 |z^2|-1).\]

Suggerimento.

Si può scrivere z in forma algebrica oppure osservare che passando l’equazione ai moduli si ottiene

    \[|z|^2 = |z|^2 (25 |z|^2 - 1),\]

da cui |z| = 0 oppure

    \[     1 = 25|z|^2 - 1 \implies |z| = \frac{\sqrt2}{5}.     \]

Nel primo caso z = 0 è l’unica soluzione possibile, mentre nel secondo si sostituisce |z| = \sqrt2 / 5 per ottenere:

    \[     z^2 = (\bar z)^2 (25 \frac2{25} - 1) = (\bar z)^2.     \]

In particolare, basta risolvere l’equazione z^2 = (\bar z)^2.

Svolgimento.

Se z = a + \imath b allora l’equazione si riscrive come

    \[(a^2-b^2)+2\imath ab = \left[ (a^2-b^2) - 2 \imath ab \right] \left( 25(a^2+b^2) - 1 \right)\]

da cui, ponendo uguali a zero parte reale e immaginaria, si ottiene il seguente sistema equivalente di equazioni reali:

    \[\begin{cases} 2(a^2-b^2) = 25 (a^2-b^2)(a^2+b^2), \\  50ab(a^2+b^2) = 0. \end{cases}\]

Dalla seconda equazione si ricava a = 0 oppure b = 0 quindi discutiamo separatamente queste due possibilità:

 

  1. Se a = 0, la prima equazione ci da

        \[-2b^2=-25b^4 \iff b^2(25b^2-2) = 0\]

    che ha come soluzioni b = 0 e b=\pm \sqrt{2}/5.

  2.  

  3. Se b = 0, invece, si trova

        \[2a^2 = 25a^4 \iff a^2(25a^2-2)=0,\]

    che ha come soluzioni a = 0 (già trovata in precedenza) e a = \pm \sqrt{2}/5.

In particolare, l’equazione di partenza ammette cinque soluzioni distinte (tre reali e due puramente immaginarie) che sono date da

    \[z_1 = 0, \quad z_{2,3} = \pm \frac{\sqrt{2}}{5}, \quad z_{4,5} = \pm \imath \frac{\sqrt{2}}{5}.\]


 
 

Esercizio 25  (\bigstar\largewhitestar\largewhitestar\largewhitestar\largewhitestar). Determinare per quali z \in \mathbb{C} si ha

    \[|z^2|(1+z^2) = 2.\]

Suggerimento.

Usare la parte immaginaria per dedurre informazioni sulle soluzioni dell’equazione.

Svolgimento.

Iniziamo osservando che se si prende la parte immaginaria dei due termini l’uguaglianza diventa

    \[\mathfrak{Im}(z^2) = 0.\]

Di conseguenza, se z=a+\imath b allora

    \[0 = \mathfrak{Im}(z^2) = 2ab,\]

e questa è soddisfatta se e solo se a = 0 oppure b = 0. È chiaro che z = 0 non è soluzione quindi supponiamo z = \imath b per b \neq 0 e sostituiamo:

    \[b^2(1-b^2)=2 \iff b^4 - b^2 + 2 = 0.\]

Poniamo t = b^2 per ridurre il grado ed osserviamo che

    \[t^2 - t + 2 = 0\]

non ammette soluzioni reali perché ha discriminante negativo (\Delta=-3). Supponiamo ora z = a per a \neq 0 e sostituiamo, ottenendo l’equazione

    \[a^2(1+a^2)=2 \iff a^4 + a^2 - 2 = 0.\]

Come sopra introduciamo una variabile ausiliaria t = a^2 ed osserviamo che

    \[t^2+t-2=0\]

ha due soluzioni reali e distinte, ovvero t_1 = 1 e t_2 = -2. La seconda non porta a nulla perché a^2 = -2 non è risolvibile in \mathbb{R}, mentre dalla prima si trova

    \[a^2 = 1 \iff a = \pm 1,\]

e quindi l’equazione di partenza ha come uniche due soluzioni z_1 = 1 e z_2=-1.


 
 

Esercizio 26  (\bigstar\largewhitestar\largewhitestar\largewhitestar\largewhitestar). Determinare per quali z \in \mathbb{C} si ha

    \[(\mathrm{e}^{2z}+4)^2 = (\imath \mathrm{e}^{2z} - 4)^2.\]

Suggerimento.

L’esponenziale complesso non si può invertire in maniera immediata come in \mathbb R, ma può essere utile sfruttare la seguente uguaglianza tra insiemi:

    \[\log z = \left\{ \text{Log }z + 2k \pi \: : \: k \in \mathbb Z\right\}.\]

Svolgimento.

Sviluppando i quadrati e portando tutto dallo stesso lato dell’equazione, si ottiene

    \[\begin{aligned} 0 & = \mathrm{e}^{4z} + 16 + 8 \mathrm{e}^{2z} - \left( \imath^2 \mathrm{e}^{4z} + 16 - 8 \imath \mathrm{e}^{2z} \right)  		\\ & = 2 \mathrm{e}^{4z} + 8(1+\imath)\mathrm{e}^{2z}. 	\end{aligned}\]

Possiamo semplificare un fattore \mathrm{e}^{2z} perché questo non è uguale a zero per alcun valore di z \in \mathbb C e dunque ci basta risolvere l’equazione

    \[\mathrm{e}^{2z} =- 4(1+\imath).\]

Poiché il logaritmo complesso è a più valori, non possiamo semplicemente invertire come in \mathbb R. Tuttavia vale la seguente uguaglianza tra insiemi:

    \[\left\{ z \in \mathbb C \: : \: \mathrm{e}^{2z} = - 4 (1+\imath)\right\} = \frac12 \log (-4 - 4 \imath).\]

Di conseguenza, ne concludiamo immediatamente che l’equazione di partenza ammette infinite soluzioni della forma

    \[z = \frac12 \text{Log }(-4-4\imath) +  k\pi \imath, \quad k \in \mathbb Z,\]

dove \text{Log } indica, come abbiamo visto, il logaritmo principale.


 
 

Esercizio 27  (\bigstar\bigstar\largewhitestar\largewhitestar\largewhitestar). Determinare per quali z \in \mathbb{C} si ha

    \[- \sin z \cos z + \cos^2 z = 1.\]

Suggerimento.

Basta applicare l’identità fondamentale della trigonometria, ovvero

    \[\sin^2 z + \cos^2 z = 1.\]

Osservazione.

Per le definizioni delle funzioni trigonometriche complesse (tramite esponenziale) e alcune proprietà analitiche, si può far riferimento a questo breve articolo.

Svolgimento.

Portiamo tutto a sinistra ed utilizziamo l’identità fondamentale della trigonometria

    \[\sin^2 z + \cos^2 z = 1\]

per riscrivere l’equazione come segue:

    \[- \sin z \cos z - \sin^2 z = -\sin z (\cos z + \sin z) = 0.\]

Il prodotto di due fattori è zero quando lo è almeno uno dei due, perciò discutiamo separatamente i due casi:

 

  1. Il primo fattore si annulla per tutti gli z \in \mathbb C tali che

        \[\sin z = 0,\]

    ma è facile vedere che questi sono dati da

        \[z = \pi k, \quad k \in \mathbb Z.\]

  2.  

  3. Per quanto riguarda il secondo fattore si può procedere direttamente, ma c’è un trucco che la semplifica notevolmente ovvero

        \[\sin \left(z+\frac\pi4\right)= \sin z \cos\frac\pi4 + \sin\frac\pi4 \cos z = \frac{\sqrt 2}{2} \left( \sin z + \cos z \right),\]

    perciò si ha l’equivalenza

        \[\sin z + \cos z = 0 \iff \sin \left(z + \frac\pi4 \right)= 0.\]

    La nuova equazione è immediata da risolvere perché, come fatto nel punto precedente, le soluzioni sono date da

        \[z + \frac\pi4 = h \pi, \quad h \in \mathbb Z\]

In conclusione, l’equazione di partenza ammette infinite soluzioni della forma

    \[z = k \pi \quad \text{e} \quad z = -\frac\pi4 + h \pi\]

al variare di k, h \in \mathbb Z.


 
 

Esercizio 28  (\bigstar\bigstar\largewhitestar\largewhitestar\largewhitestar). Determinare per quali z \in \mathbb{C} si ha

    \[(1-2\imath) \cos z + (-2+\imath) \sin z = - \sqrt{2}+\sqrt{2}\imath.\]

Suggerimento.

Sfruttare la sostituzione w:=\tan\frac{z}{2}.

Svolgimento.

Se portiamo tutto a sinistra e dividiamo per - \imath si arriva ad avere una “forma” più gestibile:

    \[(1+\imath)\sqrt{2} + (2 + \imath) \cos z - (1+2\imath) \sin z = 0.\]

A questo punto si sfrutta una sostituzione molto usata per la risoluzione di equazioni trigonometriche (reali o complesse), ovvero

    \[w := \tan \frac{z}{2},\]

in modo tale che seno e coseno si possano riscrivere, rispettivamente, come

    \[\sin z = \frac{2w}{1+w^2} \quad \text{e} \quad \cos z = \frac{1-w^2}{1+w^2}.\]

Sostituendo si trova

    \[(1+\imath)\sqrt{2} + (2+\imath) \frac{1-w^2}{1+w^2} - (1+2\imath) \frac{2w}{1+w^2} = 0,\]

da cui, imponendo la condizione w^2 \neq -1, si arriva ad avere una equazione di secondo grado nella variabile complessa w piuttosto complicata, ovvero

    \[w^2\left(-2-\imath + (1+\imath)\sqrt{2} \right) +w  \left( -2(1+2\imath) \right) + \left((1+\imath)\sqrt{2} + 2 + \imath \right)=0.\]

A questo punto è sufficiente risolvere per w ottenendo due soluzioni distinte w_1 e w_2 di cui si può trovare l’espressione esplicita. Allora si ha

    \[\tan \frac{z}{2} = w_j \implies \frac{z}{2} = \arctan w_j + \pi k, \quad k \in \mathbb{Z}\]

per j=1,2, e quindi l’equazione iniziale ha infinite soluzioni della forma

    \[z = 2\arctan w_1 + 2 \pi k \quad \text{e} \quad z = 2\arctan w_1 + 2 \pi h\]

al variare di k, h \in \mathbb Z.


 
 

Esercizio 29  (\bigstar\bigstar\bigstar\largewhitestar\largewhitestar). Determinare per quali z \in \mathbb{C} si ha

    \[z^5 + \mathfrak{Re}(\bar{z} - 7 \imath) + 4 = 0.\]

Suggerimento.

Si può scrivere z in forma algebrica e poi porre uguali a zero parte reale e immaginaria rispettivamente.

Svolgimento.

Iniziamo osservando che togliere 7 \imath alla parte reale di un numero non cambia il risultato, perciò

    \[\mathfrak{Re}(\bar{z}-7\imath)=\mathfrak{Re}(\bar{z}) = \mathfrak{Re}(z).\]

Se z = a+\imath b allora il termine a sinistra si può riscrivere come

    \[(a+\imath b)^5 + a + 4 = (a^5 - 10a^3b^2 + 5 ab^4 + a + 4) + \imath(  5 a^4 b - 10 a^2 b^3 +  b^5 ),\]

e dunque l’equazione di partenza è equivalente al sistema reale

    \[\begin{cases} a^5 - 10a^3b^2 + 5ab^4 + a + 4 = 0, \\ 5a^4b - 10a^2b^3 + b^5 = 0.\end{cases}\]

Nella seconda equazione possiamo raccogliere un fattore b ottenendo

    \[b(5a^4 - 10a^2b^2 + b^4) = 0.\]

Per trovare una decomposizione soddisfacente del secondo fattore poniamo b^2 = t, consideriamo a come un parametro ed osserviamo che

    \[t^2 - 10a^2 t + 5a^4 = 0 \implies t_{1,2} = 5a^2 \pm 2\sqrt{5}a^2,\]

da cui si trova che la seconda equazione ha cinque soluzioni:

    \[b_1 = 0, \quad b_{2,3} = \pm \sqrt{5a^2 + 2\sqrt{5}a^2}, \quad b_{4,5} = \pm \sqrt{5a^2-2\sqrt{5}a^2}.\]

Notiamo che la prima equazione del sistema dipende solo da b^2 e b^4 quindi in realtà ci son solo tre casi da discutere dato che b_3^2 = b_2^2 e b_5^2=b_4^2.

 

  1. Se b = 0 allora dobbiamo risolvere l’equazione

        \[a^5 + a+4 = 0.\]

    La funzione f(a)=a^5+a+4 ha derivata positiva, perciò f è strettamente crescente crescente. Inoltre, si ha

        \[\lim_{a \to - \infty}f(a)=-\infty \quad \text{e} \quad \lim_{a\to+\infty} f(a)=+\infty,\]

    e,di conseguenza, esiste unico a_1 \in (-3/2,-1) tale che f(a_1) = 0.

  2.  

  3. Se b = \pm \sqrt{5a^2 + 2\sqrt{5}a^2} allora si ha

        \[a^5 - 10 a^3 (5a^2 + 2 \sqrt{5}a^2) + 5a (5a^2 + 2 \sqrt{5}a^2)^2 + a + 4 = 0 ,\]

    che, ragionando come nel punto precedente, si vede ammettere un’unica soluzione a_2 \in (-1/2,0).

  4.  

  5. Se b = \pm  \sqrt{5a^2 - 2\sqrt{5}a^2} allora si ha

        \[a^5 - 10 a^3 (5a^2 - 2 \sqrt{5}a^2) + 5a (5a^2 - 2 \sqrt{5}a^2)^2 + a + 4 = 0 ,\]

    e, come già detto sopra, si vede avere una soluzione unica a_4 \in (1,3/2).

Riassumendo, l’equazione complessa di partenza ammette cinque soluzioni che sono date da

    \[z_1 = a_1, \quad z_{2,3} =a_2 + \imath b_{2,3}, \quad z_{4,5} = a_4 + \imath b_{4,5}.\]


 
 

Esercizio 30  (\bigstar\largewhitestar\largewhitestar\largewhitestar\largewhitestar). Determinare il numero di soluzioni dell’equazione

    \[\bar{z}^9 = z^3 |z|^5.\]

Suggerimento.

Utilizzare la forma esponenziale.

Svolgimento.

Sia z = \rho \mathrm{e}^{\imath \vartheta}. Allora l’equazione si riscrive come

    \[\rho^9 \mathrm{e}^{-9 \imath \vartheta} = \rho^3 \mathrm{e}^{3 \imath \vartheta} \rho^5,\]

e questa è del tutto equivalente al seguente sistema di equazioni reali:

    \[\begin{cases} \rho^9 = \rho^8, \\ -9 \vartheta= 3 \vartheta + 2k \pi. \end{cases}\]

Ovviamente z = 0 è soluzione dell’equazione di partenza, perciò se supponiamo z \neq 0 la prima ha come unica soluzione \rho = 1 e la seconda ci da

    \[12 \vartheta = 2k \pi \implies \vartheta = k \frac\pi6, \quad k \in \mathbb{Z}.\]

È facile vedere che questa ha 12 soluzioni distinte nell’intervallo [-\pi,\pi) quindi l’equazione iniziale ammette un totale di 13 soluzioni che sono date da

    \[z_0 = 0 \quad \text{e} \quad z_k = \mathrm{e}^{\imath k \frac\pi6} \text{ con }k = -6, \ldots, 5.\]


 
 

Esercizio 31  (\bigstar\bigstar\largewhitestar\largewhitestar\largewhitestar). Risolvere le seguenti equazioni nei complessi:

 

  1. 2z-4\bar{z}+|z|^2+6\imath=0;
  2.  

  3. z^2\bar{z} + z \bar{z}^2 - (3+\imath)|z|^2 - 3z^2=0.

Suggerimento.

Utilizzare la forma algebrica.

Svolgimento punto 1.

Per la prima equazione scriviamo z in forma algebrica come a + \imath b e sostituiamo, ottenendo

    \[2(a+\imath b)-4(a-\imath b) + a^2+b^2 + 6 \imath = 0.\]

Ponendo uguale a zero parte reale e immaginaria si ottiene il seguente sistema di equazioni reali:

    \[\begin{cases}-2a+a^2+b^2 = 0, \\ 6b + 6 = 0.  \end{cases}\]

Dalla seconda si trova b = -1 come unica soluzione, perciò sostituendo nella prima si trova

    \[a^2-2a + 1 = 0,\]

e questo è il quadrato di un binomio, perciò ha soluzione a=1 con molteplicità due. In particolare, l’unica soluzione di (1) è data da

    \[z = 1 - \imath.\]

Svolgimento punto 2.

Per la seconda equazione osserviamo che z \bar{z}=|z|^2 perciò si riscrive come

    \[z|z|^2 + \bar{z}|z|^2 - (3+\imath)|z|^2 - 3z^2 = 0.\]

Se z = a + \imath b si ha

    \[(a^2+b^2)(a+\imath b + (a - \imath b) - (3 + \imath)) - 3(a^2-b^2 + 2 \imath ab) = 0,\]

da cui facendo come sopra si trova il sistema

    \[\begin{cases} (2a- 3)(a^2+b^2) - 3(a^2-b^2)=0, \\ -(a^2+b^2) - 6ab = 0. \end{cases}\]

Le soluzioni di questo sistema sono (a,b)=(0,0) e

    \[ (a,b) = \left( \frac32 - \sqrt2, - \frac12 \right) \quad \text{e} \quad (a,b)= \left( \frac32 + \sqrt2, -\frac12\right). \]

Ne segue immediatamente che le soluzioni della seconda equazione sono z_1= 0,

    \[z_2 = \left( \sqrt2 + \frac32 \right) - \imath \frac12 \quad \text{e} \quad z_3 = \left( -\sqrt2 + \frac32 \right) - \imath \frac12.\]


 
 

Esercizio 32  (\bigstar\largewhitestar\largewhitestar\largewhitestar\largewhitestar). Determinare il numero di soluzioni del sistema

    \[\begin{cases}z^{10}=3+8\imath, \\ z^5 = 8 - 3 \imath.  \end{cases}\]

Suggerimento.

Non servono conti.

Svolgimento.

Poiché z^{10}=z^5 z^5 = (z^5)^2, il sistema ammette una soluzione se e solo se la seguente uguaglianza è verificata:

    \[3 + 8 \imath = (8-3\imath)^2.\]

Tuttavia, un semplice calcolo mostra che i due numeri complessi sono diversi, e conseguentemente il sistema non ammette soluzioni.


 
 

Esercizio 33  (\bigstar\largewhitestar\largewhitestar\largewhitestar\largewhitestar). Si trovino tutte le soluzioni complesse dell’equazione

    \[|\cos (\imath z) + \sinh z| =  \mathrm{e}^{\imath z}.\]

Suggerimento.

Sfruttare le definizioni di seno e coseno iperbolico per riscrivere il termine a sinistra.

Svolgimento.

Iniziamo osservando che

    \[\cos(\imath z) = \mathfrak{Re}(\imath z) = \frac{\mathrm{e}^{z} + \mathrm{e}^{-z}}{2} = \cosh z,\]

e di conseguenza il termine a sinistra dentro il modulo si può riscrivere come

    \[\cosh z + \sinh z = \frac{\mathrm{e}^{z} + \mathrm{e}^{-z}}{2} + \frac{\mathrm{e}^{z} - \mathrm{e}^{-z}}{2} = \mathrm{e}^z.\]

A questo punto, ricordando che |\mathrm{e}^z|=\mathrm{e}^{\mathfrak{Re}(z)}, l’equazione da cui siamo partiti è equivalente a

    \[\mathrm{e}^{\mathfrak{Re}(z)} = \mathrm{e}^{\imath z}.\]

Se z=x+\imath y, l’equazione si può riscrivere come

    \[ 	\mathrm{e}^x = \mathrm{e}^{-y + \imath x}, 	\]

che è del tutto equivalente al sistema reale

    \[\begin{cases} \mathrm{e}^x = \mathrm{e}^{-y}, \\ \mathrm{e}^{ix} = 1. \end{cases}\]

La prima equazione ha come soluzione x = -y, mentre la seconda equazione ha infinite soluzioni della forma

    \[ 	x = 2 \pi k \quad \text{per } k \in \mathbb Z. 	\]

Chiaramente, queste due soluzioni sono compatibili se e solo se y è un multiplo di 2\pi; in particolare, il sistema ha infinite soluzioni della forma

    \[ 	(x,y) = (2 \pi k, - 2 \pi k), \qquad \text{per } k \in \mathbb Z. 	\]

L’equazione iniziale ha infinite soluzioni

    \[ 	z_k = 2 \pi k(1 - \imath), \qquad \text{per } k \in \mathbb Z 	\]

e, di conseguenza, possiamo concludere che l’unica soluzione intera è data da z = 0.


 
 

Esercizio 34  (\bigstar\bigstar\largewhitestar\largewhitestar\largewhitestar). L’equazione z^6 + z^3 + 1 = 0 ha (una o più) radici complesse con argomento \vartheta compreso tra \pi/2 e \pi. Determinare i valori di \vartheta.

Suggerimento.

Introdurre la variabile w := z^3.

Svolgimento.

Per semplificare introduciamo la variabile ausiliaria w = z^3 e sostituiamo, ottenendo la seguente equazione di secondo grado:

    \[w^2 + w + 1 = 0.\]

Le soluzioni sono w_{1,2} = - 1/2 \pm \sqrt{3}/2 \imath, che possiamo riscrivere (per esplicitare l’argomento) in forma esponenziale come

    \[w_1 = \mathrm{e}^{\imath \frac23 \pi}\quad \text{e} \quad w_2 = \mathrm{e}^{-\imath \frac23 \pi}.\]

A questo punto possiamo trovare le soluzioni di z^3 = w_1 (per w_2 è analogo) utilizzando la forma esponenziale di z, ottenendo il sistema

    \[\begin{cases}|z|^3 = 1, \\ 3\text{Arg }z = \frac23 \pi + 2k \pi, & \text{Arg }z \in [-\pi,\pi). \end{cases}\]

La prima equazione ha soluzione |z|=1, mentre la seconda ha tre soluzioni nell’intervallo di ammissibilità che sono date da

    \[\text{Arg }z = \frac29 \pi, \quad \text{Arg }z = \frac89 \pi, \quad \text{Arg }z = - \frac49 \pi,\]

che corrispondono quindi alle soluzioni complesse

    \[z_1 = \mathrm{e}^{\imath \frac29 \pi}, \quad z_2 = \mathrm{e}^{\imath \frac89 \pi}, \quad z_3=\mathrm{e}^{-\imath \frac49\pi}.\]

Si può fare un ragionamento analogo con w_2 oppure osservare che w_2 = \bar w_1, da cui segue che le soluzioni sono date da

    \[z_4 = \bar z_1 = \mathrm{e}^{-\imath \frac29 \pi}, \quad z_5 = \bar z_2 = \mathrm{e}^{-\imath \frac89 \pi}, \quad z_6 = \bar z_3=\mathrm{e}^{\imath \frac49\pi}.\]

A questo punto è facile verificare che z_2 è l’unica soluzione che soddisfa la condizione richiesta, dunque l’unico valore di \vartheta ammissibile è 8/9 \pi.


 
 

Esercizio 35  (\bigstar\bigstar\bigstar\largewhitestar\largewhitestar). Mostrare che si possono trovare a,b \in \mathbb{C} \setminus \mathbb{R} tali che l’equazione

    \[x^2 - ax + b= 0\]

ammetta almeno una radice reale. Si dica poi se è possibile trovare a e b come sopra in modo che entrambe le radici siano reali.

Suggerimento.

La prima parte dell’esercizio si risolve semplicemente calcolando le soluzioni con la formula per equazioni del secondo grado e imponendo opportune condizioni su a,b.

Svolgimento.

Il discriminante è dato da

    \[\Delta = a^2 - 4b,\]

perciò le soluzioni dell’equazione sono

    \[x_{1,2} = \frac{a \pm \sqrt{a^2-4b}}{2}.\]

Supponiamo di voler trovare a,b \in \mathbb C in modo tale che la soluzione con il – sia reale, ovvero vogliamo che si abbia

(13)   \begin{equation*} \mathfrak{Im} \left[ a - \sqrt{a^2-4b}\right] = 0. \end{equation*}

Scriviamo a = u_a + \imath v_a e b = u_b + \imath v_b ed osserviamo subito che

    \[a^2 - 4b = (u_a^2-v_a^2 - 4 u_b) + \imath(2 u_av_a - 4 v_b).\]

Avendo bisogno della radice di questo numero, per semplificare i calcoli successivi è sensato porre la parte immaginaria uguale a zero:

    \[u_av_a = 2 v_b.\]

Segue immediatamente che

    \[\sqrt{a^2-4b}=\sqrt{u_a^2-v_a^2-4u_b},\]

ed è facile vedere che la quantità dentro la radice deve essere negativa dato che dalla condizione (13) abbiamo

    \[\mathfrak{Im} \left[ \sqrt{u_a^2-v_a^2-4u_b} \right] = \mathfrak{Im}(a) = v_a.\]

Una possibilità è quindi quella di scegliere u_a^2=4u_b (eventualmente entrambi uguali a zero, ma non è importante) e v_a > 0 così che

    \[\sqrt{a^2-4b} = \sqrt{-v_a^2} = \imath |v_a| = \imath v_a,\]

che è proprio ciò che volevamo. In particolare, segue facilmente che

    \[\frac{a - \sqrt{a^2-4b}}{2} = \frac{u_a+\imath v_a - \imath v_a}{2} = \frac{u_a}{2} \in \mathbb{R},\]

e questo conclude la prima parte dell’esercizio. Per la seconda parte supponiamo per assurdo di avere entrambe le radici reali, ovvero

    \[\frac{a + \sqrt{a^2-4b}}{2},  \frac{a - \sqrt{a^2-4b}}{2} \in \mathbb{R}.\]

Allora anche la loro somma deve essere un numero reale, ma è facile vedere che

    \[\frac{a + \sqrt{a^2-4b}}{2} +  \frac{a - \sqrt{a^2-4b}}{2} = \frac{2a}{2} = a \in \mathbb{R},\]

contro l’ipotesi da cui siamo partiti, ovvero che sia a che b devono essere numeri non reali.


 
 

Esercizio 36  (\bigstar\bigstar\bigstar\largewhitestar\largewhitestar). Risolvere al variare di a,b \in \mathbb R il seguente sistema di equazioni complesse:

    \[\begin{cases}|z|=|w|, \\ z+w = a, \\ z^2+w^2 = b. \end{cases}\]

Suggerimento.

Distinguere il caso a^2=b dal caso a^2 \neq b e, per quest’ultimo, utilizzare la forma esponenziale.

Svolgimento.

Iniziamo mostrando che nel caso a^2=b il sistema non ammette soluzioni. Infatti, eleviamo al quadrato la seconda equazione,

    \[a^2 = (z+w)^2=z^2+w^2+2zw,\]

e andiamo a sostituirla nella terza, ottenendo la relazione seguente:

    \[a^2 = b + 2zw \implies 2zw = a^2-b = 0 \implies zw = 0.\]

In particolare, uno dei due deve essere uguale a zero. Tuttavia, è facile vedere che se w=0 dalla prima equazione si trova

    \[|z|=|w| = 0 \implies z = 0,\]

e quindi

    \[a = z+ w = 0 \quad \text{e} \quad b=z^2+w^2=0,\]

contro le ipotesi che a,b \in \mathbb C sono non-nulli.

Passiamo quindi al caso a^2\neq b. La prima equazione ci suggerisce di utilizzare la forma esponenziale e, ponendo \rho = |z|=|w|, possiamo scrivere

    \[z = \rho \mathrm{e}^{\imath \alpha} \quad \text{e} \quad w = \rho \mathrm{e}^{\imath \beta}.\]

Abbiamo già utilizzato la prima informazioni, perciò si ottiene un nuovo sistema con soltanto le ultime due equazioni, ovvero

    \[\begin{cases} \mathrm{e}^{\imath \alpha} + \mathrm{e}^{\imath \beta} = \frac{a}{\rho}, \\ \mathrm{e}^{\imath 2\alpha}+\mathrm{e}^{\imath 2\beta} = \frac{b}{\rho^2}. \end{cases}\]

Elevando al quadrato la prima equazione e sostituendo nella seconda, come fatto nel caso b=a^2, si trova la relazione

    \[\mathrm{e}^{\imath(\alpha+\beta)} =\frac{a^2-b}{2\rho^2}.\]

Possiamo allora rimpiazzare una delle due equazioni con l’ultima condizione ottenuta, ricavando il nuovo sistema

    \[\begin{cases} \mathrm{e}^{\imath \alpha} + \mathrm{e}^{\imath \beta} = \frac{a}{\rho}, \\  \mathrm{e}^{\imath(\alpha+\beta)} =\frac{a^2-b}{2\rho^2}. \end{cases}\]

Questo è un sistema di secondo grado omogeneo nelle variabili \mathrm{e}^{\imath \alpha} e \mathrm{e}^{\imath \beta}, perciò per risolverlo consideriamo l’equazione ausiliaria

    \[t^2-\frac{a}{\rho} t+\frac{a^2-b}{2\rho^2}=0.\]

È semplice verificare che se t_1 e t_2 sono le due soluzioni di questa equazione ausiliaria, allora le due coppie

    \[(\mathrm{e}^{\imath \alpha},\mathrm{e}^{\imath \beta})= (t_1,t_2) \quad \text{e} \quad (\mathrm{e}^{\imath \alpha},\mathrm{e}^{\imath \beta})= (t_2,t_1)\]

sono le uniche soluzioni del sistema omogeneo. Il discriminante dell’equazione si calcola facilmente come segue:

    \[\Delta=\frac{a^2}{\rho^2}-\frac{2a^2-2b}{\rho^2}=\frac{2b-a^2}{\rho^2}.\]

Per ipotesi a e b sono numeri complessi, perciò quando prendiamo la radice del discriminante \sqrt \Delta, ci sono due possibili valori che sono le soluzioni di

    \[z^2 = \Delta.\]

È tuttavia facile mostrare che, se indichiamo con \omega una delle due soluzioni, allora l’altra sarà esattamente -\omega. Di conseguenza, si ha

    \[t_{1,2} = \frac12 \left[ \frac{a}{\rho} \pm \frac{\omega}{\rho} \right] = \frac{a \pm \omega}{2\rho},\]

il che significa che le due soluzioni del sistema sono

    \[\mathrm{e}^{\imath \alpha} = \frac{a \pm \omega}{2\rho} \quad \text{e} \quad  \mathrm{e}^{\imath \beta} = \frac{a \mp \omega}{2\rho}.\]

Affinché queste soluzioni siano accettabili dobbiamo verificare che i termini a destra dell’uguale abbiano modulo uguale ad uno o, in altre parole, che

    \[\rho=\frac{|a\pm\omega|}{2}.\]

Se moltiplichiamo le due condizioni si ottiene

    \[\rho^2=\frac{|a+\omega|\cdot|a-\omega|}{4}=\frac{|a^2-\omega^2|}{4}=\frac{|a^2-2b+a^2|}{4}=\frac{|a^2-b|}{2},\]

mentre, osservando che \rho = \rho, si ha anche l’ulteriore condizione

    \[\frac{|a+\omega|}{2}=\frac{|a-\omega|}{2} \implies |a+\omega|=|a-\omega|.\]

Elevando al quadrato quest’ultima uguaglianza e ricordando che |z|=z\bar z per ogni z \in \mathbb C, segue immediatamente che

    \[(a+\omega)(\bar a + \bar \omega) = (a-\omega)(\bar a - \bar \omega),\]

e svolgendo le moltiplicazioni e semplificando i termini comuni porta a

    \[a \bar \omega + \bar a \omega = 0.\]

Elevando al quadrato quest’ultima condizione si trova

    \[a^2 \bar \omega^2 + 2 |a|^2 |\omega|^2 + \bar a^2 \omega^2 = 0\]

e, ricordando che \omega^2 = 2b - a^2, otteniamo

(14)   \begin{equation*} 		a^2(2 \bar b - \bar a^2) + 2|a|^2 |2b-a|^2 + \bar a^2 (2b - a^2)=0. 	\end{equation*}

Ne segue perciò che il sistema di partenza ammette due soluzioni

    \[(z,w) = \left( \frac{a+\omega}{2}, \frac{a-\omega}{2}\right) \quad \text{e} \quad (z,w) = \left( \frac{a-\omega}{2}, \frac{a+\omega}{2}\right),\]

con \omega = \sqrt{2b-a^2} da intendere come detto sopra e sotto la condizione (14).


 
 

Disuguaglianze, \sup/\inf e insiemi

 

Esercizio 37  (\bigstar\largewhitestar\largewhitestar\largewhitestar\largewhitestar). Risolvere la seguente disuguaglianza

    \[\mathfrak{Re} \left( (z-1)(z-2\imath)\right) \ge \mathfrak{Re}(z-1) \mathfrak{Re}(z-2\imath).\]

Suggerimento.

Usare la forma algebrica.

Svolgimento.

Sia z = a + \imath b. I due numeri complessi si scrivono in forma algebrica come

    \[z-1 = (a-1) + \imath b \quad \text{e} \quad z - 2 \imath = a + (b-2)\imath,\]

perciò il prodotto è dato da

    \[(z-1)(z-2\imath) = a(a-1) - b(b-2) + \imath \left((a-1)(b-2) + ab \right).\]

Di conseguenza la disuguaglianza si riscrive in termini di a e b come

    \[a^2 - a - b^2 + 2b=a(a-1) - b(b-2) \ge (a-1)a = a^2 - a\]

e portando tutto a sinistra si trova

    \[-b^2+2b \ge 0.\]

La soluzione di questa disequazione è 0 \le b \le 2 perciò, considerando che

    \[ b = \mathfrak{Im}(z), \]

la disequazione di partenza è soddisfatta da tutti i numeri complessi z tali che \mathfrak{Im}(z) \in [0,2]. Graficamente si può rappresentare come una “striscia” orizzontale di spessore due.


 
 

Esercizio 38  (\bigstar\largewhitestar\largewhitestar\largewhitestar\largewhitestar). Risolvere la seguente disuguaglianza

    \[|z-2\imath|^2 - 8 > |z|^2 - |z+2\imath|^2.\]

Suggerimento.

Usare la forma algebrica.

Svolgimento.

Sia z = a + \imath b. Il termine a sinistra è dato da

    \[|z-2\imath|^2 - 8 = a^2 + (b-2)^2 - 8 = a^2 + b^2 - 4b -4,\]

mentre quello a destra

    \[|z|^2 - |z+2\imath|^2 = a^2+b^2 - a^2 - (b+2)^2 = -4b - 4.\]

La disuguaglianza si può riscrivere come

    \[a^2+b^2-4b-4 > -4b-4,\]

e se portiamo tutto a sinistra otteniamo

    \[a^2+b^2 > 0.\]

Questa è ovviamente verificata per ogni a,b \in \mathbb{R} con a,b entrambi non nulli. In altre parole, la disuguaglianza di partenza è verificata per ogni z \in \mathbb{C} \setminus \{0\}.


 
 

Esercizio 39  (\bigstar\bigstar\largewhitestar\largewhitestar\largewhitestar). Determinare l’estremo superiore e inferiore dell’insieme

    \[A = \left\{ |z-w| \: : \: |z-2| \le 1, \, \mathfrak{Re}(w-\imath \bar{w})=0 \right\}.\]

Suggerimento.

Identificare sul piano complesso le due figure geometriche date dalle condizioni per trovare z e w.

Svolgimento.

Innanzitutto osserviamo che

    \[0=\mathfrak{Re}(w-\imath\bar w) = \mathfrak{Re}(w) + \imath^2 \mathfrak{Im}(\bar w) = \mathfrak{Re}(w) - \mathfrak{Im}(w),\]

per cui w si può scrivere in forma algebrica come w = a(1+\imath), ovvero

    \[w \in \left\{ (a,b) \in \mathbb{R}^2 \: : \: a = b \right\},\]

che è esattamente la retta di coefficiente angolare uno e passante per l’origine nel piano complesso. Analogamente, la condizione

    \[|z-2| \le 1\]

ci dice che z appartiene alla palla di centro (2,0) e raggio uno nel piano complesso, quindi da un punto di vista geometrico la quantità

    \[|z-w|\]

che dobbiamo calcolare altro non è che la distanza tra due punti, uno sulla circonferenza e uno sulla retta in esame.

 

  1. L’estremo superiore dell’insieme è + \infty dato che la retta è illimitata. Infatti, dati

        \[z = 2 + 0 \imath \quad \text{e} \quad w_a := a(1+\imath),\]

    allora si ha

        \[|z-w_a|^2 = (a-2)^2 + a^2 = 2a^2 + 4 - 4a.\]

    Se prendiamo il limite per a \to +\infty, si ottiene esattamente +\infty.

  2.  

  3. Per trovare l’estremo inferiore, prendiamo un punto della retta e uno sul bordo della circonferenza, ovvero

        \[z = a + \imath \sqrt{1-(a-2)^2} \quad \text{e} \quad w = c(1+\imath).\]

    La distanza tra i due è data da

        \[|z-w|^2 = (a-c)^2 + (\sqrt{1-(a-2)^2} - c)^2\]

    ed è dunque sufficiente minimizzare rispetto ad a e c, con a\in (0,4) e c qualsiasi.

    Un approccio alternativo è il seguente. Consideriamo la famiglia di rette perpendicolari alla bisettrice del primo e terzo settore, ovvero

        \[b = - a + \kappa, \quad \kappa \in \mathbb{R},\]

    e scegliamo \kappa tale che questa passi per il centro della circonferenza:

        \[b = - a +2.\]

    Questa interseca la retta b = a nel punto w_0=1+\imath e la circonferenza nei punti che sono soluzione del sistema

        \[\begin{cases}b = - a + 2, \\ (a-2)^2+b^2=1. \end{cases}\]

    È facile vedere che le due soluzioni sono

        \[z_1 = 2 - \frac{1}{\sqrt{2}} + \frac{1}{\sqrt{2}}\imath \quad \text{e} \quad z_2 = 2 + \frac{1}{\sqrt{2}} - \frac{1}{\sqrt{2}}\imath,\]

    ma a noi interessa solo z_1 perché siamo interessati alla distanza minima tra circonferenza e retta. Si ha

        \[|z_1-w_0|^2 = \left(2 - \frac{1}{\sqrt{2}}-1\right)^2 + \left(\frac{1}{\sqrt{2}}-1\right)^2 = 3 - 2 \sqrt{2},\]

    da cui segue che

        \[\inf A = \min A = \sqrt{3-2\sqrt{2}}.\]


 
 

Esercizio 40  (\bigstar\largewhitestar\largewhitestar\largewhitestar\largewhitestar). Determinare l’estremo superiore e inferiore dell’insieme

    \[B = \left\{ |z-w| \: : \: |z+2-3\imath| \le 3, \, |w-4-4\imath|\le 4 \right\}.\]

Suggerimento.

Identificare \mathbb C con \mathbb{R}^2 ed osservare che z e w variano all’interno di due dischi per calcolare \sup ed \inf.

Svolgimento.

Come fatto nell’esercizio precedente, possiamo identificare \mathbb{C} con \mathbb{R}^2 tramite la solita corrispondenza

    \[\mathbb{C}\ni z = a+ \imath b \mapsto (a,b) \in \mathbb{R}^2.\]

Trovare l’estremo inferiore/superiore di B equivale a minimizzare/massimizzare la funzione distanza tra due palle in \mathbb{R}^2, ovvero

    \[\mathcal{C}_1 : (a+2)^2+(b-3)^2 \le 9 \quad \text{e} \quad \mathcal{C}_2 : (a-4)^2+(b-4)^2 \le 16.\]

Le due circonferenze si intersecano perciò l’estremo inferiore, che è anche un minimo, è uguale a zero. Ad esempio, il punto (a,b)=(1,3) soddisfa

    \[\begin{aligned}& (1+2)^2+(3-3)^2 = 9 \le 9 \implies (1,3) \in \mathcal{C}_1, \\ & (1-4)^2+(3-4)^2 = 10 \le 16 \implies (1,3) \in \mathcal{C}_2, \end{aligned}\]

e quindi appartiene ad entrambe le circonferenze. Per trovare l’estremo superiore consideriamo la retta che passa per entrambi i centri, ovvero

    \[r :  b = \frac{a}{6}+\frac{10}{3}.\]

Vogliamo trovare i punti di intersezione di questa retta con le due circonferenze (escludendo poi quelli più vicini) perciò iniziamo con il risolvere il sistema

    \[\begin{cases} b  = \frac{a}{6}+\frac{10}{3}, \\ (a+2)^2+(a-3)^2=9. \end{cases}\]

Questa ha come soluzioni

    \[z_1 = \frac{18}{\sqrt{37}} - 2 + \left(3 + \frac{3}{\sqrt{37}}\right)\imath \quad \text{e} \quad z_2= -\frac{18}{\sqrt{37}} - 2 + \left(3 - \frac{3}{\sqrt{37}}\right)\imath,\]

ma è facile vedere (facendo il grafico, ad esempio) che a noi interessa soltanto z_2 per massimizzare la distanza. In maniera analoga, il sistema di equazioni

    \[\begin{cases}b  = \frac{a}{6}+\frac{10}{3}, \\ (a-4)^2+(b-4)^2=16, \end{cases}\]

ha come soluzioni

    \[w_1 = 4 - \frac{24}{\sqrt{37}} + \left(4-\frac{4}{\sqrt{37}} \right)\imath \quad \text{e} \quad w_2 = 4 + \frac{24}{\sqrt{37}} + \left(4+\frac{4}{\sqrt{37}} \right)\imath,\]

ma per gli stessi motivi sopra consideriamo w_2. Allora

    \[|z_2-w_2|^2 = (7+\sqrt{37})^2,\]

da cui segue che

    \[\sup B = \max B = 7 + \sqrt{37}.\]


 
 

Esercizio 41  (\bigstar\bigstar\largewhitestar\largewhitestar\largewhitestar). Si mostri che il seguente sottoinsieme dei numeri complessi,

    \[D = \left\{ z \in \mathbb{C} \: : \: \left| \frac{z-1}{z+1} \right|= 2 \right\},\]

è una circonferenza, e se ne determini raggio e centro.

Suggerimento.

Sfruttare il fatto che il modulo di un rapporto è uguale al rapporto dei moduli.

Svolgimento.

Dalla teoria sappiamo che il modulo di un rapporto è uguale al rapporto dei moduli, perciò (assumendo z \neq -1) si ha

    \[\left| \frac{z-1}{z+1} \right| = 2 \iff |z-1| = 2 |z+1|.\]

Sia z = a + \imath b. I due moduli sono dati da

    \[|z-1|=|(a-1) + \imath b| = \sqrt{(a-1)^2 + b^2} \quad \text{e} \quad |z+1|=\sqrt{(a+1)^2+b^2},\]

e sostituendo nell’equazione troviamo

    \[\sqrt{(a-1)^2+b^2} = 2 \sqrt{(a+1)^2+b^2}.\]

Dato che entrambi i termini sono positivi per ogni valore di x ed y, possiamo elevare al quadrato ottenendo

    \[(a-1)^2 + b^2 = 4(a+1)^2 + 4b^2.\]

Portando tutto a destra otteniamo la relazione

    \[3a^2 + 10a + 3b^2 + 3 = 0,\]

che è l’equazione della circonferenza di raggio \frac43 e centro (-\frac53,0).


 
 

Esercizio 42  (\bigstar\bigstar\bigstar\largewhitestar\largewhitestar). Si caratterizzi il luogo dei punti che descrive l’insieme

    \[D = \left\{ z \in \mathbb{C} \: : \: \text{Arg } \left( \frac{z-1}{z+1} \right)= \frac\pi2 \right\}.\]

Suggerimento.

Ricordare che dati z_1,z_2 \in \mathbb{C} con z_2\neq 0 si ha

    \[\text{Arg } (z_1/z_2) = \text{Arg } z_1 - \text{Arg } z_2 + 2 \pi N_.\]

Svolgimento.

Dalla teoria sappiamo che la funzione argomento principale soddisfa la seguente proprietà

    \[\text{Arg }(z_1/z_2) = \text{Arg }z_1 - \text{Arg }z_2 + 2 \pi N_-,\]

dove N_-(z_1,z_2) è già stato definito in (3). Di conseguenza, si può caratterizzare l’insieme D trovando le soluzioni dell’equazione

    \[\text{Arg }(z-1)-\text{Arg }(z+1) + 2 \pi N_- = \frac\pi2,\]

ma c’è un modo più semplice di procedere. Infatti, posto w:=(z-1)/(z+1), l’equazione si riduce a

    \[\text{Arg }w = \frac\pi2,\]

e questa è di immediata risoluzione dato che avere argomento uguale a \pi/2 equivale ad avere parte reale zero e parte immaginaria positiva; ovvero si ha

    \[\text{Arg }w = \frac\pi2 \iff w = \imath v, \quad v > 0.\]

Per concludere ci basta esprimere parte immaginaria e reale di w in termini di z = a+ \imath b. Razionalizzando si ottiene

    \[\begin{aligned} 		w = \frac{z-1}{z+1} & = \frac{(a-1) + \imath b}{(a+1) + \imath b} \cdot \frac{(a+1)-\imath b}{(a+1)-\imath b}  		\\ & = \frac{(a-1)(a+1) + b^2 + \imath (a+1)b - \imath (a-1)b }{(a+1)^2 + b^2} 		\\ & = \frac{a^2 - 1 + b^2 + \imath (2b)}{(a+1)^2+b^2}, 	\end{aligned}\]

da cui segue che parte reale e immaginaria di w sono rispettivamente date da

    \[\mathfrak{Re}(w) = \frac{a^2+b^2-1}{(a+1)^2 + b^2} \quad \text{e} \quad \mathfrak{Im}(w) = \frac{2b}{(a+1)^2 + b^2}.\]

Per concludere, imponiamo le due condizioni trovate in precedenza, ovvero \mathfrak{Re}(w) = 0 e \mathfrak{Im}(w)>0. Si ha

    \[a^2 + b^2 - 1= 0 \qquad \text{e} \qquad b >0,\]

da cui segue che

    \[ 	D = \left\{ z \in \mathbb C \: : \: -1 < \mathfrak{Re}(z) < 1 \text{ e } \mathfrak{Im}(z) = \sqrt{1 - \mathfrak{Re}(z)^2}\right\}. 	\]


 
 

Esercizi avanzati

 

Esercizio 43  (\bigstar\bigstar\bigstar\largewhitestar\largewhitestar). Si calcolino i seguenti prodotti

    \[P_n = \prod_{k=1}^{n-1} \left( \mathrm{e}^{ \frac{2k\pi\imath}{n}} - 1 \right) \quad \text{e} \quad S_n = \prod_{k=1}^{n-1} \sin \frac{k\pi}{n}.\]

Suggerimento.

Raccogliere opportunamente i prodotti e fare alcuni tentativi per valori di n piccoli.

Svolgimento punto 1.

Per il primo prodotto osserviamo che per n=2 si ha

    \[\mathrm{e}^{\pi \imath} - 1 = - 2,\]

mentre per n=3 un calcolo immediato mostra che

    \[\begin{aligned}  \left(\mathrm{e}^{ \frac{2\pi \imath}{3}} -1 \right) \left(\mathrm{e}^{ \frac{4\pi \imath}{3}} -1 \right)&  = 1 + \mathrm{e}^{ (\frac{2}{3} + \frac{4}{3}) \pi \imath} - \mathrm{e}^{\frac{2\pi\imath}{3}}- \mathrm{e}^{\frac{4\pi\imath}{3}} 		\\ & = 2 - \mathrm{e}^{\frac{2\pi\imath}{3}}- \mathrm{e}^{\frac{4\pi\imath}{3}} 		\\ & = 2 - \mathrm{e}^{\frac{2\pi\imath}{3}} - \mathrm{e}^{-\frac{2\pi\imath}{3}}. 		\end{aligned}\]

A questo punto possiamo sfruttare l’identità \mathrm{e}^{\imath x} + \mathrm{e}^{-\imath x} = 2 \cos x che segue dalla formula di Eulero (5) ed ottenere il seguente risultato:

    \[2 - \mathrm{e}^{\frac{2\pi\imath}{3}} - \mathrm{e}^{-\frac{2\pi\imath}{3}} = 2 - \cos \frac{2}{3}\pi = 2 - \left(-\frac12\right)= 3.\]

Questi due casi ci suggeriscono che un’ipotesi sensata sul valore del prodotto è

    \[P_n = (-1)^{n-1} n,\]

e questo si può dimostrare per induzione controllando, ad esempio, se c’è un legame tra P_n e i precedenti P_k. In alternativa, osserviamo che

    \[\begin{aligned} P_n = & (-1)^{n-1} + (-1)^{n-2} \sum_{k=1}^{n-1} \mathrm{e}^{\frac{2\pi k}{n}\imath}+ (-1)^{n-3}\sum_{k = 1}^{n-1} \sum_{\ell = 1, \ell \neq k}^{n-1} \mathrm{e}^{ \frac{2\pi(k+\ell)}{n} \imath} 	\\ & + (-1)^{n-4} \cdots \end{aligned}\]

e così via fino all’ultimo termine in cui prendiamo tutti i prodotti tra gli esponenziali. Con dei semplici calcoli si può verificare che

    \[P_n = (-1)^{n-1} + (-1)^{n-1} + \cdots + (-1)^{n-1} = (-1)^{n-1} n,\]

concludendo la prima parte dell’esercizio. Per formalizzare quest’ultimo passaggio può essere utile ricordarsi i due fatti seguenti:

 

  1. I numeri complessi \mathrm{e}^{\frac{2\pi k}{n}\imath} per k = 0,\ldots,n-1 sono le radici n-esime dell’unità, ovvero le soluzioni di z^n = 1.
  2.  

  3. La proprietà dimostrata nell’Esercizio 1.15. è fondamentale per concludere.

Lasciamo al lettore di completare i dettagli della dimostrazione.

Svolgimento punto 2.

Per la seconda produttoria, invece, iniziamo osservando che

    \[S_n = \prod_{k=1}^{n-1} \sin \frac{k \pi}{n} = \prod_{k=1}^{n-1} \mathfrak{Im} \left[ \mathrm{e}^{\frac{k\pi}{n}\imath} \right],\]

e dato che la parte immaginaria si può riscrivere tramite coniugio come

    \[\mathfrak{Im}(z) = \frac{z-\bar{z}}{2\imath},\]

ne segue immediatamente che

    \[S_n =  2^{1-n} \prod_{k=1}^{n-1} \frac{ \mathrm{e}^{\frac{k\pi}{n}\imath} - \mathrm{e}^{-\frac{k\pi}{n}\imath}}{ \imath}.\]

Nel caso n = 2 abbiamo

    \[S_2 = \frac12 \cdot 2 = 1 = \sin \frac\pi2,\]

mentre per n=3 si ha

    \[\begin{aligned} S_3 &= \frac{1}{4 \imath^2} \left( \mathrm{e}^{\frac13 \pi \imath} - \mathrm{e}^{-\frac13 \pi \imath} \right) \left( \mathrm{e}^{\frac23 \pi \imath} - \mathrm{e}^{-\frac23 \pi \imath} \right) 		\\ & = - \frac14 \left[ \mathrm{e}^{\pi \imath} + \mathrm{e}^{-\pi \imath} - \mathrm{e}^{- \frac13 \pi\imath} - \mathrm{e}^{\frac13 \pi \imath} \right] 	\\ & = - \frac14 \left[ -1 -1 -1\right] = \frac34. \end{aligned}\]

Questi due casi suggeriscono l’ipotesi

    \[S_n = 2^{1-n} n, \quad n \ge 2.\]

Per dimostrarla raccogliamo un fattore

    \[\mathrm{e}^{- \frac{k\pi}{n}\imath}\]

dalla produttoria ottenendo

    \[S_n = 2^{1-n} \prod_{k =1}^{n-1} \frac{\mathrm{e}^{-\frac{k\pi}{n}\imath}}{\imath} \underbrace{\prod_{k = 1}^{n-1} \left( \mathrm{e}^{\frac{2k\pi}{n}\imath}-1\right)}_{=P_n}\]

da cui segue che

    \[S_n = 2^{1-n}n (-1)^{n-1}  \prod_{k=1}^{n-1}\frac{\mathrm{e}^{-\frac{k\pi}{n}\imath}}{\imath}.\]

A questo punto basta osservare che1

    \[\prod_{k=1}^{n-1}\frac{\mathrm{e}^{-\frac{k\pi}{n}\imath}}{\imath} = (-1)^{n-1}\]

per concludere che

    \[S_n = 2^{1-n} n (-1)^{n-1}(-1)^{n-1} = 2^{1-n} n,\]

come volevasi dimostrare.

 


    \[\]

  1. Questo fatto si può dimostrare ragionando esattamente come nell’Esercizio 1.15.

 
 

Esercizio 44  (\bigstar\bigstar\bigstar\bigstar\largewhitestar). Siano z_1,\ldots,z_n numeri complessi di modulo minore di o uguale a uno. Si mostri che esistono \epsilon_1,\ldots,\epsilon_n \in \{-1,+1\} tali che

    \[|\epsilon_1 z_1 + \cdots + \epsilon_n z_n| \le \sqrt{2}.\]

Si mostri poi che la costante \sqrt{2} è ottimale.

Suggerimento.

Verificare che, dati numeri complessi z_1,z_2,z_3 \in \mathbb{C} di modulo \le 1 esistono due indici distinti i,j \in \{1,2,3\} tali che

    \[|z_i-z_j| \le 1 \quad \text{oppure} \quad |z_i+z_j| \le 1.\]

Svolgimento.

L’ottimalità della costante \sqrt{2} è facile da vedere prendendo due vettori unitari e ortogonali, ad esempio

    \[z_1=1 \quad \text{e} \quad z_2=\imath\]

dato che per ogni scelta di segno si ha

    \[|\pm 1 \pm \imath| = \sqrt{2}.\]

Iniziamo con il dimostrare il caso n = 2 e poi cerchiamo di capire come procedere. Se z_i= x_i+\imath y_i per i=1,2, allora sappiamo che

    \[|z_i|\le 1 \implies |x_i|,|y_i| \le 1 \text{ per } i =1,2,\]

ed inoltre possiamo scegliere \epsilon_1 = 1 senza perdita di generalità. Allora

    \[z_1 + \epsilon_2 z_2 = (x_1+\epsilon_2 x_2) + \imath (y_1 + \epsilon_2 y_2),\]

da cui segue che il suo modulo al quadrato è uguale a

    \[\begin{aligned} |z_1+\epsilon_2 z_2|^2 &=  x_1^2 + x_2^2 + 2 \epsilon_2 x_1x_2 + y_1^2 + y_2^2 + 2 \epsilon_2 y_1 y_2  		\\ & = |x|^2 + |y|^2 + 2 \epsilon_2(x_1x_2 + y_1y_2) 		\\ & \le 2 + 2 \epsilon_2(x_1x_2+y_1y_2). 	\end{aligned}\]

Di conseguenza, se x_1x_2 + y_1y_2 > 0 allora si prende \epsilon_2=-1, altrimenti si sceglie \epsilon_2=1 e la disuguaglianza richiesta è verificata.

Per quanto riguarda il caso generale vogliamo far vedere che dati z_1,\ldots,z_n complessi di modulo \le 1 abbiamo la stima

(15)   \begin{equation*} |\epsilon_1 z_1 + \cdots + \epsilon_n z_n| \le \sqrt{2}, \end{equation*}

per qualche scelta di \epsilon_2,\ldots,\epsilon_n \in \{-1,1\}. Per farlo, iniziamo con il dimostrare il seguente risultato tecnico:

Lemma 2.1. Siano z_1,z_2,z_3 \in \mathbb{C} numeri complessi di modulo \le 1. Allora possiamo sempre sceglierne due (diciamo z_i,z_j) tali che

    \[|z_i-z_j| \le 1 \quad \text{oppure} \quad |z_i+z_j| \le 1.\]

    \[\]

Dimostrazione. Se per assurdo la tesi non fosse vera, allora z_1 dovrebbe avere un angolo compreso tra \pi/3 e 2\pi/3 con z_2 e con z_3. Ma allora

    \[\text{uno tra } z_2 \text{ e } - z_2 \text{ forma un angolo minore di } \pi/3  \text{ con } z_3,\]

da cui segue che uno tra z_2+z_3 e -z_2 + z_3 ha modulo minore di o uguale ad uno, che è proprio la contraddizione che cercavamo.

    \[\]

Torniamo ora alla dimostrazione della proprietà (15). Poiché n \ge 3 possiamo applicare iterativamente il lemma appena dimostrato come segue:

 

  1. Possiamo scegliere i segni \epsilon_i,\epsilon_j in modo tale che |\epsilon_i z_i + \epsilon_j z_j| \le 1, e poi rimpiazzarlo nella somma in (15) con

        \[\epsilon_{ij} z_{ij},\]

    dove z_{ij} = \epsilon_iz_i + \epsilon_j z_j soddisfa |z_{ij}|\le1 e si moltiplica per \epsilon_{ij} in caso ci sia successivamente necessità di cambiare segno.

  2.  

  3. La somma adesso contiene n-1 elementi, quindi se n-1 \ge 3 ripetiamo il primo step altrimenti possiamo passare al successivo.
  4.  

  5. Dopo k step, ovvero quando n-k = 2, la somma in (15) è rimpiazzata da

        \[\epsilon_\alpha z_\alpha + \epsilon_\beta z_\beta,\]

    con |z_\alpha|,|z_\beta|\le 1 per costruzione.

Per concludere, alla fine del processo iterativo ci riduciamo al caso n=2 che abbiamo già dimostrato in precedenza e quindi la stima (15) è verificata.

Osservazione.

Si può ottenere un risultato più preciso (che richiede però più lavoro e lasciamo al lettore interessato), ovvero dati numeri complessi

    \[z_1,\ldots,z_n \in \mathbb{C} \text{ con } |z_i| \le 1,\]

possiamo trovare \epsilon_1,\ldots,\epsilon_n \in \{-1,1\} tali che per ogni m \le n si abbia

    \[| \epsilon_1 z_1 + \cdots + \epsilon_m z_m| \le \sqrt{3}.\]

È interessante chiedersi se la costante \sqrt{3} sia effettivamente ottimale, ma al momento la risposta (per quanto ne sappiamo) non è nota.


 
 

Esercizio 45  (\bigstar\bigstar\bigstar\largewhitestar\largewhitestar). Si consideri la funzione f : \mathbb{C}\setminus\{0\} \to \mathbb{C} definita da

    \[f(z) = \frac{z^2-\bar{z}^2}{z^2}.\]

Si determini l’immagine di f e la controimmagine di un elemento w \in \mathbb{C} qualsiasi.

 

Svolgimento.

Utilizzando le coordinate polari z=\rho \mathrm{e}^{\imath \vartheta}, la funzione si può riscrivere come

    \[f(\rho,\vartheta)=\frac{\rho^2 \mathrm{e}^{\imath 2\vartheta}-\rho^2 \mathrm{e}^{-\imath 2\vartheta}}{\rho^2 \mathrm{e}^{\imath 2\vartheta}}=1-\mathrm{e}^{-\imath 4 \vartheta},\]

dove \rho \in (0,+\infty) e \vartheta \in [0,2\pi). In particolare, si ha

    \[\begin{aligned} f(\rho,\vartheta) & = 1 - \cos(-4\vartheta)-\imath \sin(-4\vartheta)  		\\ & = (1-\cos(4\vartheta)) + \imath \sin(4 \vartheta), 	\end{aligned}\]

da cui segue che f è una funzione che non dipende da \rho ed è periodica di periodo \pi/2 rispetto alla variabile \vartheta. Inoltre, si ha

    \[|f(\rho,\vartheta)|^2 = 1 + \cos^2 (4 \vartheta) - 2 \cos(4 \vartheta) + \sin^2(4\vartheta) = 2(1-\cos(4\vartheta)),\]

da cui (ricordando che 1-\cos \alpha è al più 2) segue che gli elementi nella immagine di f sono necessariamente compresi nel disco

    \[D = \{ z \in \mathbb{C} \: : \: |z| \le 2 \}.\]

Per concludere la prima parte dell’esercizio, abbiamo mostrato che

    \[\text{\rm Im} (f) = \left\{ z = (1-\cos 4 \vartheta)+ \imath \sin 4 \vartheta \: : \: \vartheta \in \left[0,\frac\pi2\right) \right\} \subset D,\]

e l’inclusione è propria perché, ad esempio, 1 \notin  \text{\rm Im} (f). Se w \in \mathbb{C} è un elemento che non appartiene all’immagine di f allora

    \[f^{-1}(w) = \emptyset,\]

altrimenti esiste \vartheta \in [0,\pi/2) tale che possiamo scrivere

    \[w = (1-\cos 4 \vartheta)+\imath \sin 4 \vartheta.\]

In tal caso, sfruttando il fatto che la funzione è periodica, abbiamo

    \[f^{-1}(w) = \left\{ z = \rho \mathrm{e}^{\imath \alpha} \: : \: \alpha = \vartheta + k \frac\pi2 \text{ con } k \in \mathbb{Z} \text{ e } \rho \in (0,+\infty) \right\}.\]


 
 

Esercizio 46  (\bigstar\largewhitestar\largewhitestar\largewhitestar\largewhitestar). Si verifichi l’identità di Lagrange complessa

    \[\left| \sum_{i= 1}^n a_i b_i \right|^2 = \left(\sum_{i=1}^n|a_i|^2\right)\left(\sum_{i=1}^n|b_i|^2\right) - \frac12 \sum_{i,j=1}^n |a_i \bar{b}_j - a_j \bar{b}_i|,\]

e se ne deduca la disuguaglianza di Cauchy-Schwarz, dove a_j,b_j \in \mathbb C.

Suggerimento.

Ricordarsi che per ogni complesso si ha |z|^2 = z \bar z e, una volta mostrato che il termine di sinistra è uguale a

    \[\sum_{i,j=1}^n a_i \bar a_j b_i \bar b_j,\]

trovare il modo giusto di raccogliere i termini della sommatoria doppia per ottenere quanto richiesto dall’esercizio.

Svolgimento.

Innanzitutto, ricordiamo che per z \in \mathbb{C} si ha

    \[|z|^2 = z \bar z,\]

perciò un semplice calcolo mostra che

    \[\begin{aligned} \left| \sum_{i=1}^n a_i b_i \right|^2& = \left(\sum_{i=1}^n a_i b_i \right)\left(\sum_{i=1}^n\bar{a}_i \bar{b}_i\right) 		\\ & = \sum_{i,j=1}^n a_i \bar{a}_j b_i \bar{b}_j 		\\ & = \sum_{i=1}^n |a_i b_i|^2 + \sum_{1 \le i < j \le n} \left( a_i \bar{a}_j b_i \bar{b}_j+a_j \bar{a}_i b_j \bar{b}_i \right) 		\\ & = \sum_{i=1}^n |a_i b_i|^2 + \sum_{i \neq j} |a_i b_j|^2 - \sum_{i=1}^n |a_i b_j|^2 + \sum_{1 \le i < j \le n} \left( a_i \bar{a}_j b_i \bar{b}_j+a_j \bar{a}_i b_j \bar{b}_i \right) 		\\ & = \sum_{i=1}^n |a_i|^2 \sum_{j=1}^n |b_j|^2 - \left( \sum_{i \neq j} |a_i b_j|^2 - \sum_{1 \le i < j \le n} \left( a_i \bar{a}_j b_i \bar{b}_j+a_j \bar{a}_i b_j \bar{b}_i \right) \right)  		\\ & = \sum_{i=1}^n |a_i|^2 \sum_{j=1}^n |b_j|^2 - \sum_{1 \le i < j \le n} \left( |a_i b_j|^2 + |a_j b_i|^2 -  a_i \bar{a}_j b_i \bar{b}_j - a_j \bar{a}_i b_j \bar{b}_i  \right) 		\\ & = \sum_{i=1}^n |a_i|^2 \sum_{j=1}^n |b_j|^2 - \sum_{1 \le i < j \le n} |a_i \bar{b}_j - a_j \bar{b}_i |^2, 	\end{aligned}\]

e questo conclude la prima parte dell’esercizio. Ora osserviamo che

    \[- \sum_{1 \le i < j \le n} |a_i \bar{b}_j - a_j \bar{b}_i |^2 \le 0,\]

da cui segue immediatamente la disuguaglianza di Cauchy-Schwarz:

    \[\left| \sum_{i=1}^n a_i b_i \right|^2 \le\sum_{i=1}^n |a_i|^2 \sum_{j=1}^n |b_j|^2.\]


 
 

Esercizio 47  (\bigstar\largewhitestar\largewhitestar\largewhitestar\largewhitestar). Si dimostri il seguente usando numeri complessi: sia ABCD un quadrilatero inscrittibile, allora si ha

    \[\overline{AC} \cdot \overline{BD} = \overline{AB}\cdot \overline{CD} + \overline{AD} \cdot \overline{BC}.\]

Questo è noto in letteratura come Teorema di Ptolemy.

Suggerimento.

Osservare che i punti non possono giacere tutti sulla stessa semicirconferenza e andare a studiare gli angoli relativi ai triangoli ABC e CDA.

Svolgimento.

Siano z_A,\ldots,z_D \in \mathbb{C} numeri complessi che si trovano, in questo ordine, sulla circonferenza di raggio unitario ovvero

    \[|z_A|,\ldots,|z_D|=1\]

e tali che non siano contenuti contemporaneamente in nessuna mezza circonferenza. Vogliamo far vedere che

    \[|z_A-z_C| \cdot |z_B-z_D| = |z_A-z_B| \cdot |z_C-z_D| + |z_A-z_D| \cdot |z_B-z_C| := (\star).\]

Si ha

    \[\begin{aligned} &\vartheta_1= \arg(z_C-z_B)-\arg(z_A-z_B)+2k\pi,\\& \vartheta_2= \arg(z_A-z_D)-\arg(z_C-z_D)+2k\pi \end{aligned},\]

dove \vartheta_1 è l’angolo relativo al triangolo ABC e \vartheta_2 quello a CDA. Poiché la somma di angoli opposti in un quadrilatero inscrittibile è \pi, si ha

    \[\begin{aligned} 0 & = \pi + \vartheta_1 + \vartheta_2 \mod 2\pi 		\\ & = \arg(-1) + \left[ \arg(z_C-z_B)-\arg(z_A-z_B)\right] + \left[\arg(z_A-z_D)-\arg(z_C-z_D) \right] 		\\ & = \arg\left[(z_A-z_D)(z_B-z_C) \right] - \arg\left[(z_A-z_B)(z_C-z_D) \right], 	\end{aligned}\]

da cui si trova la seguente uguaglianza tra insiemi:

    \[\arg\left[(z_A-z_D)(z_B-z_C) \right] = \arg\left[(z_A-z_B)(z_C-z_D) \right].\]

Scegliamo il valore principale \varphi= - \text{Arg }\left[(z_A-z_D)(z_B-z_C) \right] così che

    \[\begin{aligned} & |(z_A-z_B)(z_C-z_D)| = (z_A-z_B)(z_C-z_D) \mathrm{e}^{\imath \varphi}, 		\\ &  |(z_A-z_D)(z_B-z_C)| = (z_A-z_D)(z_B-z_C) \mathrm{e}^{\imath \varphi}.\end{aligned}\]

Di conseguenza

    \[\begin{aligned}(\star) & = \left[(z_A-z_B)(z_C-z_D) + (z_A-z_D)(z_B-z_C) \right] \mathrm{e}^{\imath \varphi} 		\\ & = (z_A-z_C)(z_B-z_D)\mathrm{e}^{\imath \varphi} 		\\ & = \left|(z_A-z_C)(z_B-z_D)\mathrm{e}^{\imath \varphi}\right| 		\\ & = |z_A-z_C| \cdot |z_B-z_D| \cdot | \mathrm{e}^{\imath \varphi} | = |z_A-z_C| \cdot |z_B-z_D| , 	\end{aligned}\]

e questo conclude l’esercizio.


 

Riferimenti bibliografici degli esercizi sui numeri complessi

[1] S. L. Parsonson, Pure Mathematics Vol. 2, Cambridge University Press, 1970.

[2] C. Mantegazza, Problemi di Analisi I dal Corso del I Anno alla Scuola Normale Superiore di Pisa, MCM, 2016.

 
 

Tutta la teoria di analisi matematica

Leggi...

  1. Teoria Insiemi
  2. Il metodo della diagonale di Cantor
  3. Logica elementare
  4. Densità dei numeri razionali nei numeri reali
  5. Insiemi Numerici \left(\mathbb{N},\, \mathbb{Z},\, \mathbb{Q}\right)
  6. Il principio di induzione
  7. Gli assiomi di Peano
  8. L’insieme dei numeri reali: costruzione e applicazioni
  9. Concetti Fondamentali della Retta Reale: Sintesi Teorica
  10. Costruzioni alternative di \mathbb{R}
  11. Binomio di Newton
  12. Spazi metrici, un’introduzione
  13. Disuguaglianza di Bernoulli
  14. Disuguaglianza triangolare
  15. Teoria sulle funzioni
  16. Funzioni elementari: algebriche, esponenziali e logaritmiche
  17. Funzioni elementari: trigonometriche e iperboliche
  18. Funzioni goniometriche: la guida essenziale
  19. Teorema di Bolzano-Weierstrass per le successioni
  20. Criterio del rapporto per le successioni
  21. Definizione e proprietà del numero di Nepero
  22. Limite di una successione monotona
  23. Successioni di Cauchy
  24. Il teorema ponte
  25. Teoria sui limiti
  26. Simboli di Landau
  27. Funzioni continue – Teoria
  28. Il teorema di Weierstrass
  29. Il teorema dei valori intermedi
  30. Il teorema della permanenza del segno
  31. Il teorema di Heine-Cantor
  32. Il teorema di esistenza degli zeri
  33. Il metodo di bisezione
  34. Teorema ponte versione per le funzioni continue
  35. Discontinuità di funzioni monotone
  36. Continuità della funzione inversa
  37. Teorema delle contrazioni o Teorema di punto fisso di Banach-Caccioppoli
  38. Teoria sulle derivate
  39. Calcolo delle derivate: la guida pratica
  40. Teoria sulle funzioni convesse
  41. Il teorema di Darboux
  42. I teoremi di de l’Hôpital
  43. Teorema di Fermat
  44. Teoremi di Rolle e Lagrange
  45. Il teorema di Cauchy
  46. Espansione di Taylor: teoria, esempi e applicazioni pratiche
  47. Polinomi di Taylor nei limiti: istruzioni per l’uso
  48. Integrali definiti e indefiniti
  49. Teorema fondamentale del calcolo integrale (approfondimento)
  50. Integrali ricorsivi
  51. Formule del trapezio, rettangolo e Cavalieri-Simpson
  52. Teoria sugli integrali impropri
  53. Funzioni integrali – Teoria
  54. Introduzione ai numeri complessi – Volume 1 (per un corso di ingegneria — versione semplificata)
  55. Introduzione ai numeri complessi – Volume 1 (per un corso di matematica o fisica)
  56. Serie numeriche: la guida completa
  57. Successioni di funzioni – Teoria
  58. Teoremi sulle successioni di funzioni
    1. 58a. Criterio di Cauchy per la convergenza uniforme
    2. 58b. Limite uniforme di funzioni continue
    3. 58c. Passaggio al limite sotto il segno di integrale
    4. 58d. Limite uniforme di funzioni derivabili
    5. 58e. Piccolo teorema del Dini
    6. 58f. Procedura diagonale e teorema di Ascoli-Arzela
  59. Serie di funzioni – Teoria
  60. Serie di potenze – Teoria
  61. Serie di Fourier – Teoria e applicazioni
  62. Integrali multipli — Parte 1 (teoria)
  63. Integrali multipli — Parte 2 (teoria e esercizi misti)
  64. Regola della Catena — Teoria ed esempi.
  65. Jacobiano associato al cambiamento di coordinate sferiche
  66. Guida ai Massimi e Minimi: Tecniche e Teoria nelle Funzioni Multivariabili
  67. Operatore di Laplace o Laplaciano
  68. Teoria equazioni differenziali
  69. Equazione di Eulero
  70. Teoria ed esercizi sulla funzione Gamma di Eulero
  71. Teoria ed esercizi sulla funzione Beta
  72. Approfondimento numeri complessi
  73. Diverse formulazioni dell’assioma di completezza
  74. Numeri di Delannoy centrali
  75. Esercizi avanzati analisi

 
 

Tutte le cartelle di Analisi Matematica

Leggi...

  1. Prerequisiti di Analisi
    1. Ripasso algebra biennio liceo
    2. Ripasso geometria analitica
    3. Ripasso goniometria e trigonometria
    4. Errori tipici da evitare
    5. Insiemi numerici N,Z,Q,R
    6. Funzioni elementari
    7. Logica elementare
    8. Insiemi
  2. Successioni
    1. Teoria sulle Successioni
    2. Estremo superiore e inferiore
    3. Limiti base
    4. Forme indeterminate
    5. Limiti notevoli
    6. Esercizi misti Successioni
    7. Successioni per ricorrenza
  3. Funzioni
    1. Teoria sulle funzioni
    2. Verifica del limite in funzioni
    3. Limite base in funzioni
    4. Forme indeterminate in funzioni
    5. Limiti notevoli in funzioni
    6. Calcolo asintoti
    7. Studio di funzione senza derivate
    8. Dominio di una funzione
    9. Esercizi misti Funzioni
    10. Esercizi misti sui Limiti
  4. Funzioni continue-lipschitziane-holderiane
    1. Teoria sulle Funzioni continue-lipschitziane-holderiane
    2. Continuità delle funzioni
    3. Continuità uniforme
    4. Teorema degli zeri
    5. Esercizi sul teorema di Weierstrass senza l’uso delle derivate
  5. Calcolo differenziale
    1. Derivate
    2. Calcolo delle derivate
    3. Retta tangente nel calcolo differenziale
    4. Punti di non derivabilità nel calcolo differenziale
    5. Esercizi sul teorema di Weierstrass con l’uso delle derivate
    6. Studio di funzione completo nel calcolo differenziale
    7. Esercizi teorici nel calcolo differenziale
    8. Metodo di bisezione
    9. Metodo di Newton
  6. Teoremi del calcolo differenziale
    1. Teoria sui Teoremi del calcolo differenziale
    2. Teorema di Rolle
    3. Teorema di Lagrange
    4. Teorema di Cauchy
    5. Teorema di De L’Hôpital
  7. Calcolo integrale
    1. Integrale di Riemann
    2. Integrali immediati
    3. Integrale di funzione composta
    4. Integrali per sostituzione
    5. Integrali per parti
    6. Integrali di funzione razionale
    7. Calcolo delle aree
    8. Metodo dei rettangoli e dei trapezi
    9. Esercizi Misti Integrali Indefiniti
    10. Esercizi Misti Integrali Definiti
  8. Integrali impropri
    1. Teoria Integrali impropri
    2. Carattere di un integrale improprio
    3. Calcolo di un integrale improprio
  9. Espansione di Taylor
    1. Teoria Espansione di Taylor
    2. Limiti di funzione con Taylor
    3. Limiti di successione con Taylor
    4. Stime del resto
  10. Funzioni integrali (Approfondimento)
    1. Teoria Funzioni integrali (Approfondimento)
    2. Studio di funzione integrale
    3. Limiti con Taylor e De L’Hôpital
    4. Derivazione di integrali parametrici (Tecnica di Feynmann)
  11. Numeri Complessi
    1. Teoria Numeri complessi
    2. Espressioni con i numeri complessi
    3. Radice di un numero complesso
    4. Equazioni con i numeri complessi
    5. Disequazioni con i numeri complessi
    6. Esercizi misti Numeri complessi
  12. Serie numeriche
    1. Teoria Serie numeriche
    2. Esercizi Serie a termini positivi
    3. Esercizi Serie a termini di segno variabile
    4. Esercizi Serie geometriche e telescopiche
  13. Successioni di funzioni
    1. Teoria Successioni di funzioni
    2. Esercizi Successioni di funzioni
  14. Serie di funzioni
    1. Teoria Serie di funzioni
    2. Esercizi Serie di funzioni
  15. Serie di potenze
    1. Teoria Serie di potenze
    2. Esercizi Serie di potenze
  16. Serie di Fourier
    1. Teoria Serie di Fourier
    2. Esercizi Serie di Fourier
  17. Trasformata di Fourier
    1. Teoria Trasformata di Fourier
    2. Esercizi Trasformata di Fourier
  18. Funzioni di più variabili
    1. Teoria Funzioni di più variabili
    2. Massimi e minimi liberi e vincolati
    3. Limiti in due variabili
    4. Integrali doppi
    5. Integrali tripli
    6. Integrali di linea di prima specie
    7. Integrali di linea di seconda specie
    8. Forme differenziali e campi vettoriali
    9. Teorema di Gauss-Green
    10. Integrali di superficie
    11. Flusso di un campo vettoriale
    12. Teorema di Stokes
    13. Teorema della divergenza
    14. Campi solenoidali
    15. Teorema del Dini
  19. Equazioni differenziali lineari e non lineari
    1. Teoria equazioni differenziali lineari e non lineari
    2. Equazioni differenziali lineari e non lineari del primo ordine omogenee
  20. Equazioni differenziali lineari
    1. Del primo ordine non omogenee
    2. Di ordine superiore al primo,a coefficienti costanti,omogenee
    3. Di ordine superiore al primo,a coefficienti costanti,non omogenee
    4. Di Eulero,di Bernoulli,di Clairaut,di Lagrange e di Abel
    5. Non omogenee avente per omogenea associata un’equazione di Eulero
    6. Sistemi di EDO
  21. Equazioni differenziali non lineari
    1. A variabili separabiliO
    2. A secondo membro omogeneo
    3. Del tipo y’=y(ax+by+c)
    4. Del tipo y’=y(ax+by+c)/(a’x+b’y+c’)
    5. Equazioni differenziali esatte
    6. Mancanti delle variabili x e y
    7. Cenni sullo studio di un’assegnata equazione differenziale non lineare
    8. Di Riccati
    9. Cambi di variabile: simmetrie di Lie
  22. Analisi complessa
    1. Fondamenti
    2. Funzioni olomorfe
    3. Integrale di Cauchy e applicazioni
    4. Teorema della curva di Jordan e teorema fondamentale dell’Algebra
    5. Teorema di inversione di Lagrange
    6. Teorema dei Residui
    7. Funzioni meromorfe
    8. Prodotti infiniti e prodotti di Weierstrass
    9. Continuazione analitica e topologia
    10. Teoremi di rigidità di funzioni olomorfe
    11. Trasformata di Mellin
  23. Equazioni alle derivate parziali
    1. Equazioni del primo ordine
    2. Equazioni del secondo ordine lineari
    3. Equazioni non-lineari
    4. Sistemi di PDE
  24. Funzioni speciali
    1. Funzione Gamma di Eulero
    2. Funzioni Beta,Digamma,Trigamma
    3. Integrali ellittici
    4. Funzioni di Bessel
    5. Funzione zeta di Riemann e funzioni L di Dirichlet
    6. Funzione polilogaritmo
    7. Funzioni ipergeometriche
  25. Analisi funzionale
    1. Misura e integrale di Lebesgue
    2. Spazi Lp,teoremi di completezza e compattezza
    3. Spazi di Hilbert,serie e trasformata di Fourier
    4. Teoria e pratica dei polinomi ortogonali
    5. Spazi di Sobolev
  26. Complementi
    1. Curiosità e approfondimenti
    2. Compiti di analisi
    3. Esercizi avanzati analisi
  27. Funzioni Convesse

 
 

Tutti gli esercizi di geometria

In questa sezione vengono raccolti molti altri esercizi che coprono tutti gli argomenti di geometria proposti all’interno del sito con lo scopo di offrire al lettore la possibilità di approfondire e rinforzare le proprie competenze inerenti a tali argomenti.

Strutture algebriche.


 
 

Risorse didattiche aggiuntive per approfondire la matematica

Leggi...

  • Math Stack Exchange – Parte della rete Stack Exchange, questo sito è un forum di domande e risposte specificamente dedicato alla matematica. È una delle piattaforme più popolari per discutere e risolvere problemi matematici di vario livello, dall’elementare all’avanzato.
  • Art of Problem Solving (AoPS) – Questo sito è molto noto tra gli studenti di matematica di livello avanzato e i partecipanti a competizioni matematiche. Offre forum, corsi online, e risorse educative su una vasta gamma di argomenti.
  • MathOverflow – Questo sito è destinato a matematici professionisti e ricercatori. È una piattaforma per domande di ricerca avanzata in matematica. È strettamente legato a Math Stack Exchange ma è orientato a un pubblico con una formazione più avanzata.
  • PlanetMath – Una comunità collaborativa di matematici che crea e cura articoli enciclopedici e altre risorse di matematica. È simile a Wikipedia, ma focalizzata esclusivamente sulla matematica.
  • Wolfram MathWorld – Una delle risorse online più complete per la matematica. Contiene migliaia di articoli su argomenti di matematica, creati e curati da esperti. Sebbene non sia un forum, è una risorsa eccellente per la teoria matematica.
  • The Math Forum – Un sito storico che offre un’ampia gamma di risorse, inclusi forum di discussione, articoli e risorse educative. Sebbene alcune parti del sito siano state integrate con altri servizi, come NCTM, rimane una risorsa preziosa per la comunità educativa.
  • Stack Overflow (sezione matematica) – Sebbene Stack Overflow sia principalmente noto per la programmazione, ci sono anche discussioni rilevanti di matematica applicata, specialmente nel contesto della scienza dei dati, statistica, e algoritmi.
  • Reddit (r/Math) – Un subreddit popolare dove si possono trovare discussioni su una vasta gamma di argomenti matematici. È meno formale rispetto ai siti di domande e risposte come Math Stack Exchange, ma ha una comunità attiva e molte discussioni interessanti.
  • Brilliant.org – Offre corsi interattivi e problemi di matematica e scienza. È particolarmente utile per chi vuole allenare le proprie capacità di problem solving in matematica.
  • Khan Academy – Una risorsa educativa globale con lezioni video, esercizi interattivi e articoli su una vasta gamma di argomenti di matematica, dalla scuola elementare all’università.






Document









Document








Document









Document